סיקור מקיף

החומר האפל זורק רמז / קלרה מוסקוביץ

לאחר עשרות שנים של ממצאים שליליים, הפיזיקאים מצאו סוף-סוף קצה חוט

גלקסיות מתנגשות בצביר אָבֶּל 3827. באדיבות המצפה האירופי הדרומי (ESO), נאס"א וסוכנות החלל האירופית/טלסקופ החלל האבל
גלקסיות מתנגשות בצביר אָבֶּל 3827. באדיבות המצפה האירופי הדרומי (ESO), נאס”א וסוכנות החלל האירופית/טלסקופ החלל האבל

יש משהו אי שם ביקום. אנחנו לא יכולים לראות אותו, אנחנו לא יכולים לגעת בו ואנחנו יודעים שהוא שם רק בזכות המשיכה הכבידתית שהוא מפעיל על העצמים ביקום. זה עשרות שנים, הסיפור של החומר האפל היה תגלית אחר תגלית בנוגע למה שהחומר המסתורי הזה איננו, סינון הדרגתי של אפשרויות שגרם לפיזיקאים להרגיש עצבנות גדלה והולכת. מה יקרה כשהמועמד האחרון יימחק מן הרשימה? האם ייחרץ גורלנו, לא לזכות לעולם להציץ בטיבו של החומר שתורם כ-25% מסך כל המסה ביקום?

הכתמים הכחולים מראים אור מכופף, שהאסטרונומים משתמשים בו כדי לשקול ארבע גלקסיות מתמזגות.

בתסריט העגום הזה חלה תפנית מעוררת תקווה בתחילת אביב 2015. חוקרים חשפו את אחד הרמזים המסקרנים ביותר זה שנים: סימן לקיומו של כוח חדש שעשוי לאפשר לחומר האפל “לדבר” עם עצמו. התובנה הזאת תעזור להסביר מאיזה סוג חלקיקים עשוי, אולי, החומר האפל.

הרמז הזה עלה בתצפיות שנערכו על פינה ביקום המכונה צביר אָבֶּל 3827. אסטרונומים איתרו לאחרונה את מיקומו של חומר אפל בתוך ארבע גלקסיות מתנגשות בצביר הזה על ידי שימוש בתופעה המכונה עידוש כבידתי (כיפוף האור כשהוא עובר סמוך לעצמים בעלי מסה). תצפיות שנערכו באמצעות טלסקופ החלל האבל ובאמצעות “הטלסקופ הגדול מאוד” בצ’ילה חשפו שהחומר האפל המקיף לפחות אחת מן הגלקסיות האלה משתרך מאחורי החומר הרגיל המצוי שם. התצפית רומזת שחלקיקי החומר האפל מנהלים אינטראקציות אלו עם אלו ומאטים את עצמם: תופעה שמעולם לא נראתה בעבר.

צוות אסטרונומים שבראשו עומד ריצ’רד מאסי מאוניברסיטת דרהם שבאנגליה משערים שמכיוון שהאינטראקציות לא השפיעו על החומר הרגיל, הן כנראה התרחשו באמצעות כוח אחר, שאינו כבידה, כוח המשפיע רק על חומר אפל. חילופי “פוֹטונים אפלים”, למשל, עשויים ליצור את הכוח הזה. מצב כזה עשוי להיות מקביל לאופן שבו פרוטונים רגילים מנהלים אינטראקציות אלו עם אלו באמצעות הכוח האלקטרומגנטי: כששני פרוטונים מתקרבים זה לזה, כל אחד מהם משחרר פוֹטון, נושא הכוח של האלקטרומגנטיות, והאחר קולט אותו. החילופין האלה מעבירים תנע, וגורמים לשני הפרוטונים להיפרד זה מזה.

החדשות האלה ריגשו את הפיזיקאים ועוררו אותם לחפש תשובות. “אם התגלית הזאת תחזיק מעמד, אז זה הרבה מעבר לחדשות מרעישות,” אומר הפיזיקאי ניל ויינר מאוניברסיטת ניו יורק, שלא נטל חלק במחקר. תרחיש הכולל פוטונים אפלים הוא שינוי של התפיסה הבסיסית והמקובלת ביותר של חומר האפל כמורכב מסוג אחד ויחיד של חלקיקים, שמקובל לכנותו “חלקיק מסיבי המנהל אינטראקציות חלשות”, או בראשי תיבות WIMP (“חלשלוש”, באנגלית). אבל הרעיון שלחומר אפל מתלווים פוטונים אפלים ואינטראקציות אקזוטיות עשוי לעזור לפתור כמה בעיות שמעורר תיאור החומר האפל באמצעות WIMP מסוג אחד, כמו למשל מדוע מרכזי הגלקסיות פחות דחוסים מן הצפוי.

התפיסה הזאת עשויה גם לעזור לפיזיקאים לצמצם במידה רבה את רשימת המועמדים לחומר האפל. “אף על פי שיש לנו ראיות לקיומו של חומר אפל ממגוון עצום של מקורות,” אומר ויינר, “נכון לעכשיו אין לנו שום דבר שמורה בבירור על משהו מעבר לאינטראקציות הכבידתיות שלו. אם יתברר שיש לו אינטראקציות עצמיות ברמה הזאת, נוכל לבטל מספר עצום של מודלים” שמתארים מה עשוי להיות החומר האפל. בייחוד, הממצא הזה, לאינטרנט שהתפרסם באופן מקוון באפריל ובדפוס ביוני 2015 בכתב העת Monthly Notices of the Royal Astronomical Society, עשוי לסתור גרסאות נפוצות רבות של השערה הגורסת כי החומר האפל הוא חלקיק שנחזה על ידי תיאוריית הסופר-סימטריה. התיאוריה הזאת, רעיון מפתה שמנסה להסביר תעלומות רבות בפיזיקה, כמו למשל מדוע המסה של בוזון היגס נמוכה כל כך, מניחה שיש ביקום עוד חלקיקים שעדיין לא התגלו. ואולם, אם אחד מן החלקיקים האלה (שעשוי להיות WIMP) הוא האחראי לחומר האפל, אז מרבית הגרסאות של התיאוריה אינן חוזות אינטראקציות עצמיות.

המחברים הנוספים של המאמר אומרים שמוקדם מדי לבטל הסבר יום-יומי יותר לתצפיות שלהם. לדוגמה, ייתכן שחומר אפל המצוי מחוץ לגלקסיות המתנגשות, אבל לאורך קו הראייה מכדור הארץ, עשוי לתרום לעידוש הכבידתי. “אחד הסייגים במחקר החדש הזה הוא שמדובר רק בעצם אחד,” אומר חבר הצוות דייוויד הארווי מן המכון הטכנולוגי הממשלתי של שווייץ שבעיר לוזאן. “איננו יודעים מה איננו יודעים, והגורמים הבלתי ידועים האלה עלולים להשפיע על התוצאה.” בנוסף, חיפושים קודמים בצבירים אחרים לא גילו סימנים לחומר אפל המנהל אינטראקציות עם עצמו, בין השאר מחקר שהתפרסם במארס 2015 בכתב העת סיינס, שבראשו עמד הארווי ושניתח 72 התנגשויות של צבירי גלקסיות, ולא גלקסיות יחידות. ואולם, צבירים מתנגשים מהר יותר מגלקסיות, ולכן יש פחות זמן לחומר האפל לנהל אינטראקציות ולפגר מאחור, כך ששני הממצאים אינם סותרים זה את זה.

אם יתברר שהתצפיות האחרונות אינן משקפות כוחות חדשים או אינטראקציות חדשות של חומר אפל, אָבֶּל 3827 יהפוך לעוד דוגמה למה שהחומר האפל איננו. בינתיים, החיפושים אחר החלקיקים שלו בגלאים תת-קרקעיים ממשיכים להעלות חרס. בנוסף, החומר האפל טרם הופיע במאיץ ההדרונים הגדול (LHC) של CERN. המדענים מקווים שהמגמות האלו ישתנו בקרוב: המאיץ חזר לפעול באפריל 2015, באנרגיות הגבוהות ביותר שהגיע אליהן אי פעם, והגלאים כעת הם רגישים להפליא. “החומר האפל היה כל כך חמקמק עד כה, אבל אף פעם לא היו בידינו הנתונים שעומדים להגיע אלינו,” אומר הארווי. “לתחושתי, זה עכשיו או לעולם לא.”

235 תגובות

  1. ישראל שפירא
    תאמין לי קראתי מספר ספרים בנושא. אותם הבנתי. אותכם לא. אפשר להבין דף גמרא אותכם לא. יש לכם כישרון יוצא מהכלל לקחת נושא שמספיק לכתוב עליו פיסקה, שתיים לכל היותר ואתם. לוקחים עט מרוקנים את כול הדיו. ריצ’ארד פיינמן שקיבל פרס נובל לא היה מבין אתכם. אפשר לכתוב על נושא שבעולם מורכב ככל שיהיה בצורה עניינית וקצרה. את זה אתם לא יודעים לעשות. חבל. נראה שרוח הפוסט מודרניזם שורה עליכם. בזה אני מסיים את הוויכוח שלי אתכם.

  2. אריק וישראל שפירא
    לי אין שום יומרות, לכם כן וכך אתם נראים. מסתובבים במי אפסיים של פוטונים וקוורקים וחושבים שאתם מבינים. טעות בידכם. הבוזונים של היגנס עוד יטריפו עליכם את דעתכם שגם כך לא בטוח שהיא צלולה. שתיתם הרבה מאוד וודקה.

  3. נו גורילון, ולאיזו פריצת דרך מדעית הגעת אתה? קילוף בננה באמצעות הזנב? ואיזה פרס קיבלת? גביע הקוקוס?

    אולי תסביר לנו אתה כיצד הארץ מושכת את הירח? עם החבל של טרזן?

    האנושות עוצרת את נשימתה.

    גם חיות הג’ונגל.

    קדימה!

  4. גורילה יקרה
    אנחנו ס”ה כולנו קופיי האדם שמנסים להבין דברים ולהחליף חוויות. אחרי שאנו מעלים כל אחד בתורו את הסבריו אנו מבינים את החומר יותר טוב. ומכל מלמדי השכלתי!
    שבוע טוב
    ורד קצת מהעץ
    יהודה

  5. לכל המגיבים למיניהם. אולי תפסיקו לשחק את התפקיד של פילוסופים בגרוש. אף אחד מכם לא בא איזו שהיא פריצת דרך מדעית. לא קיבלתם ולא תקבלו פרס נובל. אתם נעים במעגלים כמו כלב שרץ אחרי הזנב שלו. אולי תעשו מה שהוא פרודוקטיבי יותר כמו קריאת ספר. איך אומרים,השעמום הוא אם כל חטאת. ממרום שנותי אני מרחם עליכם.

  6. לפוצץ ביצה, לאיזה אלוהים אתה מתכוון?
    לבודהה? לזאוס? לאודין? אולי לאללה?
    במהלך ההסיטוריה האנושית הכתובה בני האדם האמינו בכ1000 אלוהים שונים. תבחר אחד ואשמח אם תנמק את בחירתך.

  7. elbentzo,
    קבל בבקשה את התנצלותי.
    לפני כמה שנים ניסיתי לפרוש את משנתי כאן באתר הידען בדרך של תגובות ושיחות עם חברים. יצאתי מנקודת התחלה של “יקום מכלום” וניסיתי להראות כי “כלום” הוא ישות (ליתר דיוק, אינות) בעלת תכונה פעילה אחת – שלילה. ניסיתי להסביר איך מן האין נוצר יש ללא התערבות חיצונית (מסוג אלהים שהזכיר טמבלצו). אפילו גייסתי לעזרה משחקי מילים (לא-לא נותן כן, אבל כן-כן לא נותן לא). אולם תמיד נתקלתי בחומת אי-הבנה מן הסוג “אבל היכן רותמים את הסוס”, כגון “מה המסה של חלקיק היוצר מסה?”. העליתי טענה כי כפי שהמספר “הדמיוני” i הנו הכרח המציאות, קיים גם ערך אמת לוגי שהחלתו על עצמו נותנת FALSE; איננו מכירים אותו במציאות היומיומית, אך קיומו פותר לנו את בעיית חוסר הסימטריה בלוגיקה ומראה כי שני ערכי האמת המוכרים, TRUE & FALSE, נובעים ממנו, כמו גם פתרון לשאלה מדוע המרחב שאנחנו פועלים בו הוא תלת-מימדי.
    לרגע היה נדמה לי שכבר ביקרת כאן …

  8. יובל
    הדבר היחיד שלא צריך להסביר בעולם הזה זה את אלוהים.
    הוא כבר הסביר את עצמו היטב.
    לנו רק נשאר לנסות להבין אותו למה הוא התכוון.
    🙂

  9. יובל,

    1. אני לא צוחק עליך.

    2. אין לי מושג מי אתה ומעולם לא קראתי שום דבר שכתבת (אלא אם כן פרסמת אותו ב-JHEP או משהו כזה, למרות שנראה לי שאז הייתי זוכר את שמך).

    3. אתה לא היחיד שרצה להגדיר מבנה מתמטי אבסטרקטי שמתכונותיו המתמטיות בלבד ינבעו כל התופעות הפיזיקליות. עשו את זה לפניך. למעשה, תורת שדות היא בדיוק תורה כזאת. אין בה אף חלקיק, אין בה הגדרה של מסה או של כוח. כל מה שיש זה מרחב וקטור (Fock Space), עליו מוגדרות סדרה של העתקות בעלי תכונות סימטריה מסוימות, ואקום (במובן המתמטי, לא במובן של ריק), וכו’. מהגדרות אלה אנו *מקבלים* את החלקיקים, מקבלים את המושג של כוחות ושל התמדה, מקבלים מה זה מסה וכו’.

  10. לפחות לי, זה נשמע כמו המלט.
    ואני לא מדבר על המחזה.
    אני מדבר על המאכל שמכינים מביצים.
    למה שלא נלך רחוק יותר?
    – אלוהים המציא וקבע את החלקיקים?

  11. elbentzo
    אתה צוחק עלי. נכון?
    אינני יודע מי מסתתר מאחורי הכינוי שלך (אני יכול רק לנחש), אבל אני חושד שיצא לך לקרוא פרקים מן המודל שלי 🙂
    אכן, אני מתחיל במבנה מתמטי, חסר כל תכונות פיזיקליות, ודרך ניתוח מתמטי בלבד אני מגיע למבנים מורכבים שאותם אני מייחס לחלקיקי החומר האפל.

  12. עקב תופעת העידוש המקושרת למסה (שהוכחה בתצפיתו של אדינגטון) הניחו ההוגים הפיסיקאים כי גלים אלקטרומגנטיים במופעם החלקיקי (פוטונים) ניחנים במסה. היות שלא נמדדה מסה כזאת המציאו את המושג “מסת תנועה”.
    תופעת העידוש היא רק אחת ממספר תופעות המקושרות למסה. שתים אחרות ידועות הן גרביטציה והתמדה. אלה שלוש תכונות שונות במהותן, ואותי מעניין לדעת מהו המנגנון המייצר אותן ותופעות אפשריות נוספות.

  13. יובל,

    למה שלא נלך צעד אחד רחוק יותר? עזוב אותך מחלקיקים. בוא נגדיר רק מבנה מתמטי, חסר כל תכונות פיזיקליות, שלפי המבנה שלו (בעיקר הסימטריות אבל לא רק) יקבעו אילו חלקיקים יש. כלומר, אפשר להסתכל על פיזיקה שבה חלקיקים הם בכלל לא דברים יסודיים. איך זה נשמע לך?

  14. יודה, ישראל
    לבוזון היגס יש מסה כתוצאה מאינטרקציות שיש בתוך שדה היגס (אם אני מבין נכון).
    ואל תשכחו גם שיש דבר כזה “אנרגיה אפלה”. אולי היא “נותנת” מסה לחלקיקי חומר אפל? או חלקיקי פושינג גרביטי?
    אוף… כבר נהיה לי הפרשי לחצים במוח..

  15. יודה

    אני לא חכם ממך,אך אכן שאלתי שאלה זו. ע”ע “שבירת סימטריה” עלק.

    אך במודל מכני פשוט כמו פושינג, איני רואה כיצד החלקיק שמעניק מסה לגופים יהיה בעל מסה בעצמו.

    יש מבין?

  16. לישראל
    אני לא יש מבין, ואני אשאל את השאלה שבטח שאלו כבר חכמים ממני: אם הבוזון היגס נותן את המסה למי שאין לו מסה אז מי נותן את המסה לבוזון היגס?? הבוזון היגס עצמו?…. אז אם הוא יכול, למה כל אחד לא יכול לתת לעצמו את המסה?? . יכול להיות שניתקל פה בפרדוקס ידוע, ” פרדוקס הספר” שאומר: אם הספר מספר את כל מי שאינו מסתפר בעצמו אז מי מספר את הספר???
    חומר למחשבה.
    יאללה יש היום הרצאה מעניינת באגודה בגבעתיים: שמש גרעינית. צריך להתכונן.
    ערב טוב
    סבדרמיש יהודה

  17. ישראל ויהודה (הממלכה המאוחדת) 🙂

    ישראל: מסה מתבטאת גם בעידוש

    יהודה: אין לי בעיה עם חלקיקי פושינג גרביטי. לא מן הנמנע כי הם אכן מקיימים ומבצעים את כל מה שאתה מטיל עליהם. יש להם מסה, יש להם תנע, אנרגיה קינטית וכל מה שצריך כדי לייצר פיסיקה. הם רק לא יסודיים מספיק לטעמי. אני מחפש ישות המשוללת כל אותן תכונות פיסיקליות בסיסיות. כי, אחרת, לא התקדמנו כלל.
    אזרום איתך: בוא נבנה מודל המבוסס על חלקיק חסר תכונות פיסיקליות אשר מן ההתנהגות שלו – כיחיד או כקבוצה – נוצרים חלקיקי פושינג גרביטי.

  18. יודה, מסה מתבטאת בשתי תכונות:

    1. התנגדות להאצה (אינרציה).

    2. משיכה גרביטציונית.

    בכל המדידות שנערכו עד כה, אין הבדל מדיד בין 2 תכונות אילו של מסה, לכן נוכל לראות אותן כתכונה אחת.

    בפושינג גרביטי, ללא החלקיקים לא קיימת מסה גרביטציונית. הארץ והשמש יכולים להיות במרחק מטר זה מזו ללא כל משיכה ביניהם. לכן מהנאמר בסעיף הקודם, גם לא קיימת מסה אינרציאלית ללא החלקיקים.

    ולכן, אם לגופים אין מסה ללא החלקיקים, כיצד יתכן שלחלקיקים עצמם תהיה מסה? אילו חלקיקים קטנים יותר מעניקים להם את מסתם? בוזוני היגס לייט?

    ?

    ??

    ??!?

  19. אני לא מבין מה זה חלקיק בלי מסה להבנתי המושג חלקיק אם ורק אם מסה. נכון, אני יודע שלמשל לפוטון אין מסת מנוחה ועוד כמה הגדרות משונות, אבל אני לא צריך להתנצל בגלל שלחלקיק שלי יש מסה. ואם אני אגדיר שיש לו תנע זה כן יספק אותך?, מה לגבי אנרגיה קינטית, מותר שיהיה לו?, לחלקיק שלי יש את כל מה שצריך לפושינג גרביטי ואם יש להיגס בוזון יש בעיה עם זה אז זבש”ו (זו בעיה שלו)
    נא להגיב בעדינות
    יום טוב
    יהודה

  20. יודה, אם לחלקיקי הפושינג גרביטי יש מסה והם גם מקור המסה של כל החומר הבריוני, אז מה מעניק מסה לחלקיקי הפושינג גרביטי? חלקיקים קטנים יותר? שבתורם מקבלים את מסתם מחלקיקים עוד יותר קטנים? שבתורם… חלקינקונים… חלקינקונוקונים… שבתורם..

  21. יהודה,
    אצלי אין לו מסה. אם אנחנו מגדירים מסה באמצעות מסה הרי לא לא הרווחנו כלום.
    מסה היא ביטוי לתופעה הנגרמת מן ההתנהגות של אוסף חלקיקי חומר אפל.

  22. מעניין יובל, גם בפושינג גרביטי רבים טוענים שהחלקיקים של הפושינג גרביטי, מרכיבים את החומר הבריוני ואצלך אותו דבר רק שהם אפלים אולי בסופו של דבר זו אותה גברת בשינוי אדרת??, חומר למחשבה.
    הדגש צריך להיות מה שאתה אומר: ” להגדיר תחילה את תכונותיו המרחביות והמתמטיות של חלקיק החומר האפל. ” סוף ציטוט.או במילים אחרות…. להגדיר את החלקיק של הפושינג גרביטי. אצלי מסתו היא בגודל של 6.3 כפול עשר בחזקת מינוס 37 גרם, מה אצלך??
    לילה טוב
    יהודה

  23. תודה אבי
    העיקרון העומד מאחורי המודל שהגיתי לפני 40 שנה ושהיום אני מייחס אותו לחומר האפל נשען על עקרון הפשטות.
    הפיסיקה מייצגת היום “גן חיות” גדול של חלקיקים וכוחות אשר מצד אחד נראים שונים זה מזה אך מצד שני “מדברים” זה עם זה. רובם חולקים תכונות מסוימות, והראשונה בהן היא התכונה מסה. חלוקה אחת שאני עושה בהקשר לתכונה הזאת, מסה, היא אלה שזו נמדדה בהם ואלה שזו לא נמדדה בהם או שנמדדה רק בעקיפין.
    היקום מאוכלס בחלקיקי חומר אפל בצפיפות המשתנה ממקום למקום. בצפיפות גבוהה הם יוצרים מבנים שאנו מכירים כחומר בריוני. בצפיפות נמוכה הם יוצרים מה שאנו מכנים חלל ריק. בחומר הבריוני אנו יכולים למדוד מסה. בחלל הריק מתאפשרת התפשטות של גלים אלקטרומגנטיים.
    כדי לענות על השאלה כיצד מסתדרים חלקיקי החומר האפל ליצירת מבנים בריונים מצד אחד וחלל ריק מצד שני וכיצד נבנים חוקי הפיסיקה בכלל, יש להגדיר תחילה את תכונותיו המרחביות והמתמטיות של חלקיק החומר האפל. אך צריך להיזהר לא לתת לו איזשהן תכונות פיסיקליות ידועות, היות שהוא אמור לבנות את הפיסיקה הידועה ולא להיבנות ממנה.

  24. תודה אריאל. שיניתי את אתר הבית כהצעתך ואני מחכה בדחילו ורחימו לראות מה יקרה.

    תודה גם לך, יהודה. בלי להיכנס לפרטי המודל שלך אומר רק שאינני אוהב את הפרשי הלחצים שעליהם אתה מדבר. זאת, מן הסיבה הפשוטה שלחץ הוא תופעה פיסיקלית המוגדרת כפונקציה של גודל פיסיקלי בסיסי יותר (כוח), וגם הוא מוגדר כפונקציה של של גודל פיסיקלי בסיסי יותר (מסה); ובאופן הזה מתקבלת הגדרה מעגלית שאינה מובילה אותנו לשום מקום.

  25. רפאל
    שאלה טובה! כשאנחנו מסתכלים על גלקסיה אחרת, כמו אנדרומדה, גודל הגלקסיה קטנה לעומת המרחק. לכן, ניתן להניח שכל האור מהגלקסיה מגיע באותו זמן.

    בנוסף, הזמן הוא קטן יחסית לקצב השינוי של מצבם היחסי של הכוכבים. אז, אם נסתכל בגלקסיה שלנו, טעות של עשרות אלפי שנים אינה גורמת לטעות מהותית במיקום הכוכבים.

  26. לאריאל
    זה לא חייב להיות המעריך של החזקה אלו יכולים להיות אינ סוף אפשרויות אחרות. למשל קבוע הגרביטציה G עלול להיות שונה במקצת (אינסוף אפשרויות), וכו, במרחקים קרובים בדקנו וגילינו שנוסחת ניוטון עם החזקה 2 של המרחק מתאימה לנו אבל זה רק עד אלפית שנת אור מעבר לכך כל ניסיון לשימוש בנוסחה מראה שיש חריקות בנוסחה, למשל אנומליה של פיוניר, מסה אפלה, ועוד.
    ליובל חייקין
    אמרת:- ” עדיין אין ידע רב לגבי תכונותיו של חומר אפל, אך יש ודאות לגבי קיומו. זאת, על סמך שתי תופעות שנצפו: עידוש גרביטציוני ואנומליה בתנועת כוכבים בקצוות גלקסיות.”. סוף ציטוט.
    אמרת שניצפה עידוש גרביטציוני, ואתה טועה כי מה שצפית בו הוא רק עידוש, גרביטציוני או לא זה רק ההסבר שלך. יש עוד הרבה סוגי עידוש למשל עידוש שנוצר ע”י עדשה או הפרש לחצים .. לגבי האנומליה בתנועת כוכבים בקצה הגלקסיה, אז הויכוח המתנהל כאן. הוא מה זה ולמה זה קורה?
    ערב טוב כאן בישראל
    יהודה

  27. יובל אני חושב ששינוי הקישור יפתור את הבעיה. תשנה לדף הבית של הידען והתגובות שלך לא יצטרכו לחכות לשחרור(הסיבה היא מניעת פרסום)

  28. יש לי שאלה שמנקרת במוחי כבר זמן רב. אולי כבר כולם יודעים את התשובה חוץ ממני. כאשר אנחנו מסתכלים על היקום, התמונה שאנחנו רואים אינה תמונת מצב של נקודה אחת מסויימת בזמן אלה כל נקודה שייכת לזמן אחר. לדוגמה כאשר אנחנו מסתכלים על גלקסיה אז יש כוכבים הקרובים אלינו יותר ויש כאלה הרחוקים מאותם כוכבים קרובים עשרות שנות אור ויותר. איך ניתן לקבוע באופן כזה שהגלקסיה אכן בצורת ספירלה או כל צורה אחרת הניראת לנו?

  29. יהודה אתה עדיין לא הסברת את הכוונה במרחקים גדולים… מה אתה טוען? שככל שהמרחק יותר גדול כך המעריך יקטן?(המעריך ינתן כפונקציה של המרחק)
    או שאתה מתכוון שהמעריך קבוע אבל הוא שונה מ2?

  30. אבקת חשמל אלקטרופיזית, שיהיה…
    את תגובתי זו אני מביא גם ביקום המקביל, היות ששם אין צורך להמתין לשחרור. לא יזיק לך לפתוח חשבון פייסבוק. זה בחינם.

    “מרחקים גדולים”? מרחק הוא מרחק בכל גודל ואין שום סיבה לקבוע כי במרחקים מסוימים תפעל פיסיקה אחרת מאשר במרחקים אחרים.

    עדיין אין ידע רב לגבי תכונותיו של חומר אפל, אך יש ודאות לגבי קיומו. זאת, על סמך שתי תופעות שנצפו: עידוש גרביטציוני ואנומליה בתנועת כוכבים בקצוות גלקסיות.

    איך נוצרים הגוף והגרביטציה מתוך החומר האפל או מדוע אני טוען שגוף אינו מייצר גרביטציה? ובכן, התכונה מסה מתבטאת לפחות בשני אופנים שונים ובלתי תלויים זה בזה, אשר תורת היחסות הכללית טוענת שהם שני פנים של אותה תכונה – התמדה ומשיכה – וגם חוזה בהצלחה את תופעת העידוש הגרביטציוני. קיום שני אופנים שונים של אותה תכונה מצריך חיפוש המנגנון הגורם לשניהם וגם את התופעה הנוספת שנחזתה (ואוששה בידי אדינגטון).
    על פי המודל שפיתח ניוטון, כוח המשיכה בין שני גופים הוא מתכונתי למכפלת המסות שלהם ומתכונתי הפוך לריבוע המרחק ביניהם. בהעדר ידע נוסף קל לקפוץ למסקנה כי המסות של הגופים הן היוצרות את כוח המשיכה ביניהן, וכך אכן נוהגים הפיסיקאים עד היום. גם תופעת ההתמדה מתכונתית למסה וכמוה תופעת העידוש. בגלל הדמיון לתופעות פיסיקליות אחרות הועלתה ההשערה כי הגרביטציה היא תופעה דואלית – גלית/חלקיקית – הנישאת על פני תווך לא מוכר באמצעות חלקיקים הנקראים “גראביטונים”, אך אותם חלקיקים לא גויסו, לפי שעה, להסביר גם את תופעת ההתמדה ואת העידוש. נכון להיום טרם התגלו גראביטונים וגם טרם התגלה המנגנון המייצר את התופעות השונות המיוחסות למסה. גילויו של החומר האפל מעורר תקווה כי הגילוי המיוחל יגיע בקרוב.

    לא רק אתה משחק פוקר בימי שלישי ולא רק יהודה גר ברחוב אלי כהן בהרצליה.

  31. למר אבקת חשמל
    אנסה להסביר לך למה הכוונה במרחקים גדולים
    בנוסחת הגרביטציה של ניוטון יש גורם R המביע את המרחק בין שני הגופים הנימשכים גרביטציונית.מרחקים קטנים זה מערכת השמש הפנימית עד ל60 יחידות אסטטרונומיות שזה בערך עשרה מיליארד ק”מ(כאלפית שנת אור)
    נוסחת הגרביטציה של ניוטון מחשבת למרחק זה נתונים הזהים כמעט למציאות. קטן אפילו אם יהיה מדובר בשני כוכבים באנדרומדה.
    אבל נניח ש R הוא 60,000 יחידות אסטרונומיות – כ- 10,000 מיליארד ק”מ שזה כשנת אור אחת , האם גם כעת נוכל להיות בטוחים שנוסחת ניוטון תיתן ניבויים נכונים?. וזה עדין נחשב למרחק
    ומה לגבי R שהוא המרחק ממרכז הגלקסיה שלנו עד למרכז גלקסית אנדרומדה כשני מיליון שנות אור שהם 20 מיליון מיליון מיליון ק”מ , זה כבר מרחק גדול.האם גם אז נהיה בטוחים שנוסחת ניטון תהיה נכונה למרחק זה? כלומר האם גם אז היא תראה לנו את כוח המשיכה הגביטציוני המדוייק בין שביל החלב לאנדרומדה?, ומה לגבי מרחקים של מיליארדי שנות אור????
    הבנת את זה מר פודרה אלקטרונית?

    יום טוב
    סבדרמיש יהודה

  32. ישראל, יובל, יודה
    תודה. אך עדיין לא הבנתי את הכוונה של האמירה: “במרחקים גדולים”?

    ערקמן
    אחלה תסרוקת. 🙂

    אז אתה טוען – כפי המודל הסטנדרטי – שהגרביטציה זה כוח נפרד. עומד בפני עצמו, כמו הכוח החלש למשל. הבנתי אותך נכון?
    שנית, איך נוצרים הגוף והגרביטציה מתוך החומר האפל? צפידות החלקיקים? 🙂 אשמח אם תסביר.
    הרי עדיין אין ידע לגבי תכונותיו של חומר אפל, מה גם שאפילו ודאות לגבי קיומו עדיין מוטלת בספק רב. כל מה שידוע הוא שיש עידוש ושזה נובע – כנראה – בגלל עוד חומר כלשהו. אפל.

    עוד דבר: מדוע אתה טוען שגוף אינו מייצר גרביטציה? האם המסה של גוף לא יוצרת כוח כלשהו המושך אליו גוף בעל מסה נמוכה יותר?
    אולי זה רק אני אבל תמיד חשבתי שגופים יוצרים כוח שנקרא כוח משיכה, רק ש עדיין לא יודעים מהם החלקיקים המרכיבים את הכוח הזה.
    טוב נו… שלישי. פוקר.
    אלך לי לגינה לקטוף כמה עגבניות אפלות בנות 80 שנה ואשלח אותם לרחוב אלי כהן בהרצליה בעילום שם, תוך כדי תקווה שאקבל בחזרה 80 גרם של רחת לוקום.
    דרך אגב, יובל, אשמח אם תגיב כאן מכיוון שאיני מנוי בפיסבוק או טויטר וכל זה.

  33. אלבנטזו,
    בהמשך לשיחתנו, הציטוטים הבאים מהבלוג של קארול באים להסביר מדוע אני תוהה לגבי מהו *צופה* (וגם עבור אחרים שאולי יתעניינו)
    they’re due to an effective measurement of the quantum state of the inflaton field when the universe reheats at the end of inflation

    But when we’re not looking, a system in its ground state (like an electron in its lowest-energy orbital around an atomic nucleus) isn’t fluctuating at all; it’s just sitting there

    but there are no particle detectors around, so in fact the quantum fields are sitting there quietly in a stationary state with no definite particle number. Therefore, these kinds of fluctuations aren’t “really happening.”

    ואז קראתי את זה:
    the microscopic states of the plasma provide an environment that becomes entangled with the large-scale fluctuations of the inflaton, effectively measuring it and collapsing the wave function

    האם זה אומר שכל ״אלמנט״ קלאסי הוא בעצם צופה? האם אני יכול להסיק שכל מה שהוא אינו קלאסי לא יכול להיות צופה שיגרום לקריסת פונקציית הגל?

    אשמח אם תבקר מחדש את רעיון יקום מכלום שכן, אם אני מבין נכון, קארול אכן מדבר על פרטובציות אלא שהוא טוען שהן התרחשו בסיום שלב האינפלציה (ואם אני מבין נכון, זה מצב בו יש כבר מבנים קלאסיים שמשמשים כצופים) אבל ביקום מתנפח שיש בו רק אנרגיית וואקום ואין צופים, אין פלקטואציות קוונטיות בכלל (זה מה שהוא כתב. אני הוספתי את נושא הצופים). מדוע שזה יהיה שונה במצב של ייקום קר (היט דת׳)
    אני מבין שהוא מתבסס על יקומים מרובים אבל תחת הנחה זו, אם הוא ועמיתיו צודקים, האם בכל זאת יש כאן בעיה לרעיון יקום מכלום?

  34. יהודה,
    ההוריקן המפלצתי נוצר היכן שרוב הרוחות מרוכזות יכנו היכן שרוב המסה נמצאת. בבולט קלסטר הכבידה נמצאת לא היכן שרוב החלקיקים שאנחנו מכירים נמצאים. איך אתה מסביר זאת?
    מדוע אתה מתנגד לחומר אפל (שאינך יודע מהן בכלל) ע״י הצגת חומר אפל אחר, סופר בעייתי שקל להפריכו בניסוי פשוט כפי שאלבנטזן הציע? איך אתה לא רואה את זה?

  35. שלומי נרקיס הוא שם שבדיתי. לתומי סברתי כי השם יובל חייקין מדליק נורה אדומה. עוד אפענח את תעלומת החסימה הסלקטיבית. בכל מקרה, אני מגיב גם במאמר המקביל בפייסבוק שזה הקישור אליו
    https://www.facebook.com/hayadan/posts/1026656190686882

    שמוליק,
    חלקיקי החומר האפל והרווחים שביניהם הם אלה הבונים את החומר מחד, ומהווים את התווך בו מתפשטים הגלים האלקטרומגנטיים מאידך. היחס הכמותי בין החלקיקים והרווחים (צפיפות) הוא הקובע את גודל מבני החומר ואת מהירות העברת הגלים האלקטרומגנטיים. גוף מסיבי נוצר מצפיפות גבוהה של חלקיקי חומר אפל. מפל ריכוזים מביא לצפיפות גבוהה גם בסביבת הגוף המסיבי, וזה מה שמאט את תנועת הגלים האלקטרומגנטיים ומביא להתעקמות האור.

    אלכימיה כבר יצאה מן האופנה ומודל החומר האפל שאני מביא מתואם עם החוק השני של התרמודינמיקה.
    בסופו של דבר זה לא יהיה מאד מעניין.

  36. יהודה

    אתה כותב “החלקיקים עצמם מסתובבים מסביב לשמש כמו כל חומר אחר”.

    יופי, אבל באיזה כיוון? ובאיזו מהירות?

    היתכן שלכל גוף שסובב את השמש מותאמים חלקיקים משלו? ומה עם הירח שסובב את הארץ? וחללית אפולו שסובבת את הירח? ודפנה שסובבת את אפולו ומסובבת לו את הראש?..

    אך אל יפול רוחך, אח שלנו. בלה בנעימים עם הילדים והנכדים.

    קיים פיתרון אלגנטי לבעיית החיכוך, אך אין זאת אומרת שפושינג גרביטי הוא הסבר אמיתי.

  37. ניסים
    רוצה הסבר אחר מלבד גרביציה?,,,, בבקשה
    שני הוריקנים נעו ברחבי האקיינוס אחד כלפי השני והתנגשו ונוצר הוריקן יותר גדול- מקרה אמיתי שאני זוכר מידע אישי. גרביטציה לא עבדה כאן. הפרש לחצים? כן!
    ניראה לי שזה אפשרי גם לגבי גלקסיות,
    אז זהו אני קופץ לנכדים
    ביי
    יהודה

  38. יהודה
    אנחנו יודעים מתצפיות שגלקסיות מתנגשות, וגם רואים בצילומים את ההשפעה של הכבידה על הצורה של הגלקסיות.

    אשמח לשמוע הסבר אחר לצילומים אלה.

  39. שמוליק ואריאל
    יכול להיות שאתם צודקים והחומר האפל קיים ואיתה האנרגיה העפה לה וגם הסברתי עשר פעמים את עניין מהירות האור אז בואו ניפרד כידידים פשוט אני עייף כמעט שבוע רק מגיב ועומד תחת אש ואפלה ויש לי לסיים עבודה ללקוח שממתין לה מחר וגם הנכדים המקסימים שלי הגיעו. אז היה נחמד ואפילו נעים
    כל טוב ביי ולהתראות ונא לא לכעוס.
    יהודה

  40. אריאל,

    אתה טוען שאתה לא פיזיקאי, אבל אולי היית צריך להיות. בקצרה, עלית בול על הטעות של יהודה (טוב נו, אחת מן הטעויות הרבות מאוד מאוד מאוד של יהודה) וגם על איך מודדים אותה ומראים לו בניסוי שהוא טועה (אם הוא יקשיב לך זו כבר שאלה אחרת, והניחוש שלי הוא כמובן – לא).

    מכיוון שאני רק חוזר על מה שכבר אמרת, אתמצת בשתי שורות – מה שיהודה מתאר זה התפשטות של חומר בתוך מרחב אינסופי, ולא התפשטות של מרחב. אלה דברים שונים בתכלית. האופן להבדיל ביניהם הוא לבדוק האם אנו מרגישים תאוצה. גוף בתאוצה יודע שהוא מאיץ (זה גודל מדיד), ואנו יודעים בפירוש שאנחנו *לא* מאיצים החוצה מאיזה מרכז של היקום (בתיאור של יהודה חייב להיות מרכז לצביר הגז, כלומר להתפלגות החומר ביקום, ואנו יודעים שזה לא המצב), אלא שהיקום מתרחב בכל נקודה באופן איזוטרופי ושהתרחבות זו נהיית מהירה יותר. זה לא תאוצה במובן של גוף מאיץ (שפועל עליו כוח), אלא מרחב שכל המרחקים בו מתארכים בקצב גובר.

    אבל למרות ההבחנה היפה של אריאל, יש משהו יותר בסיסי שאפשר למדוד במעבדה תיכונית, וזה מה שכתבתי בתגובה הקודמת. כל מה שצריך לעשות כדי להפריך את התורה המכנית (ואכן זו הדרך הראשונה שבה הפריכו אותה, למרות שיש אוסף ענק של הפרכות) זה לזרוק כדור בחדר בלי התנגדות אוויר ולבדוק אם נוצר חיכוך יש-מאין באופן מסתורי.

  41. יהודה,
    לא איכפת לי במה תשתמש אלא שתורת היחסות מנבה בהצלחה כל כך הרבה דברים כולל איסור של חומר עם מסה לחצות את מהירות האור. לפיכך, אתה לא רואה את הבעייתיות של ההצעה שלך?

    לא ענית איך הרעיון שלך מסביר את הבולט קלסטר???

    לא ענית איך אתה לא רואה את הסתירה הקטגורית בלקדם חומר אפל תוך צעקות מכאן ועד הודעה חדשה, שאין חומר אפל.
    בהגדרה, אתה מדבר על חלקיקים שעוד לא גילינו. זה ח ו מ ר א פ ל !!!!

  42. יש לך כמה טעויות שכנראה נובעות מחוסר הבנה.
    1)ההתפשטות שאתה מתאר היא התפשטות של חומר לא של מרחב וזה ההתפשטות של היקום.
    2)כוכב שנמצא במנוחה לא ירגיש תאוצה הוא יראה כאילו שאר הכוכבים מתרחקים ממנו והסיבה היא שוב, ההתפשטות היא של המרחב בין הכוכבים כלומר נוצר מרחב חדש ביניהם.
    3) התאוצה של היקום נמצאת במגמת עלייה. היא לא משחקת יויו. יש משהו שנקרא קבוע האבל והוא אומר שעל כל מיליון פרסק(פרסק = 3.26 שנות אור) מהירות העצם ביחס אלינו תראה גדולה בעוד 69.32 ק”מ לשנייה.
    כלומר ככל שהמרחק גדול יותר המהירות תגדל עוד יותר.
    4)בסופו של דבר ההתפשטות שלך תגיע למצב של שיווי משקל עם כבידה(בין אם אתה צודק בנוגע למעריך של המרחק ובין אם אתה לא) ויווצר יקום סטטי.

  43. אריאל
    הפרש הלחצים שלי מסביר יפה יפה את ההאצה בהתפשטות היקום בלי צורך באנרגיה אפלה. כל כך פשוט שבא לבכות!
    אז נא לחדד את החושים כי אני לא אסביר פעמיים. ובכן, נערוך ניסוי:-
    אל תוך מיכל וואקום ענק בקוטר אינסופי הכנס כדור גאזי בקוטר 100 שנות אור שבתוכו מפוזרים כוכבים. בתחילה תתפשט אל תוך הריקנות שיכבה מספר מאה- החיצונית ביותר שכבה מספר 99 לא תנוע כלל כי המידע על הריקנות יגיע רק אחרי שנה ולא יכול לנוע יותר ממהירות האור. אחרי שנה תתחיל לנוע גם שכבה 99 הפנימית יותר ואחרי עוד שנה שכבה מספר 98 ואחרי עשר שנים שכבה מספר 90. עד כאן אתה מסכים אריאל? אם אתה מסכים אז אתה מסכים שכוכב בשיכבה פנימית לא נע, אבל לאחר שנים מסויימות הוא יתחיל לנוע במהירות גוברת והולכת כלפי חוץ- וזה פרושו תנועה בתאוצה. לא צריך אנרגיה אפלה ולא בטיח!
    ולא רק זה, זה מסביר גם את אופי התאוצה בהתחלה היא תלך ותגדל ולאחר מכן תלך ותיקטן אבל תמיד תהיה תאוצה .
    אז הנה , כולם אומרים שיש אנרגיה אפלה ואפילו הרבה, ואני יהודה סבדרמיש אומר שאין כלום.
    אז בימים אלה של ירידות שערים בכל הבורסות בעולם אני לא הייתי משקיע במסה או באנרגיה אפלות.
    הערב האפל יורד אז
    ביי לכולם
    יהודה

  44. יהודה אומנם אני לא יודע אם רק מסה/אנרגיה אפלה הן הפתרון היחיד ורוב הסיכויים שלא אבל אני לא פיזיקאי ולכן אני אחכה עם זה. הפרש הלחצים שלך פשוט לא יכול להסביר את האצה בקצב התפשטות היקום מהסיבה הפשוטה :
    לחץ של גז תלוי במספר גורמים: בצפיפות הגז(יח’ מסה/ליח’ נפח) טמפרטורה (משוואת הגזים האידאליים).
    עכשיו לפי הטענה שלך הפרש הלחצים מסביר את האצה בקצב התפשטות היקום אבל ככל שהיקום גדל כך הנפח גדל, היקום מתקרר, והלחץ של הגזים שלך יורד מה שאמור להראות ירידה בתאוצה של ההתפשטות מה שלא עומד עם תוצאות הניסויים שמראה שקצב ההתפשטות תלוי במרחק(אנרגיה אפלה מסבירה זאת).

  45. נקודה צפה ושמוליק
    ניראה לי שהפושינג גרביטי של לה סאז’ שונה מהפושינג גרביטי שלי. היקום הפשוט והגזי שלי מדבר על חלקיק קטן שגודלו פחות מעשר בחזקת מינוס 36 גרם ומהירותו רק כמהירות האור. לגבי מהירות האור היקרה. עשיתי חישוב ניטוני וקיבלתי פי 1.3 ממהירות האור. ואני יודע שמהירות האור היא מקסימלית ולכן קרוב לוודאי שהמהירות היחסותית צריכה להיות קצת יותר קטנה ממהירות האור אבל האם מותר לי להשתמש בתורת היחסות כאשר אני בודק רעיון זר?, זה כמו שאני אשתמש במכונית שלך רק בגלל שיחשבו שאנחנו חברים. בצדק יכלו לטעון שאסור לי להשתמש בתורת היחסות. אז השארתי את זה פתוח ויחליט כל אחד מה מתאים לו.
    ניסים
    מאיפה לקחת את הקביעה הזאת?. מה שאני טוען שעל סמך הפושינג גרביטי שלי הכולל גם יקום גזי מה שניראה כהתנגשות ודאית שלנו עם אנדרומדה לא בהכרח יהיה כך. השאלה אם היקום גזי או לא. ודרך אגב אם כבר הזכרת זאת אז יכול להיות שגלקסיה שמכילה המון חומר אפל הנעה בסיבוב בתוך חומר אפל, גם אצלה יהיה אפקט מגנוס!!!. נמתין 4 מיליארד שנים ניראה ונחליט!
    יום טוב
    יהודה

  46. יהודה,

    הראתי לך כבר לפני שנים רשימה של בעיות שאין להן הסבר ללא חומר אפל. אפילו נתתי לך מספר קישורים למאמרים מדעיים הנוגעים לראיות פוזיטיביות לקיום החומר האפל. תמיד סירבת לקרוא אותן.

    רוצה עוד פעם? בבקשה. הראה לי כיצד ללא חומר אפל או אנרגיה אפלה אתה מסביר את ה-power spectrum של קרינת הרקע הקוסמית (בפרט את מבנה העליות והמורדות שלה).

  47. יהודה,

    נראה שאתה מתעקש לא להבין את הנושא הפשוט מאוד של החיכוך. באחת התגובות האחרונות אמרת שאולי צריך ניסוי. אז הנה, יש לי ניסוי בשבילך.

    בוא נקח חדר עם וואקום על פני כדה”א. בחדר הזה נירה כדור מתותח במקביל לקרקע (להלן, ציר איקס). לפי תורת היחסות הכללית (וכבידה ניוטונית, שהיא מקרה פרטי שלה בקירוב של שדה חלש), לא יפעל אף כוח על ציר איקס. על ציר ווי (רדיאלי ממרכז כדה”א), לעומת זאת, יהיה כוח בקירוב קבוע עם תאוצה של בערך 10 מטר לשניה בריבוע. כלומר, התיאוריה הזאת חוזה מסלול מסוים – מסלול פרבולי מושלם.

    התיאוריה המכנית של הכבידה גם חוזה מסלול מסוים, אבל אחר. למה? בנוסף לכוח הכבידה שהיא חוזה (שבמקרה הזה הוא זהה לכבידה הניוטונית, או בלשונך – מרחקים קצרים), היא חוזה גם חיכוך בציר איקס בניגוד לכיוון התנועה. הסבר מילולי לכך מצוי בצורה מאוד יפה ופשוטה בהרצאה של פיינמן שקישרת אליה – כדור התותח נעה בכיוון מסוים ובכיוון התנועה שלו הוא מתנגש בהרבה יותר חלקיקים מאשר בכיוון הנגדי (כי הוא רודף אחריהם ולא בורח מהם, כמו בגשם). לכן הפרש הלחצים שאתה כל כך אוהב ידחוף אותו חזרה לתוך התותח.

    זה ניסוי פשוט שאפשר לעשות בכל מעבדה. למעשה, כשאני הדרכתי מעבדות בתור מאסטרנט באוניברסיטה אי-שם-פה-בסביבה, העברתי ניסוי מאוד דומה. עכשיו אני אתן לך לנחש – כשעושים את הניסוי הזה על פני כדה”א, האם מוצאים שכדור התותח נע על ציר איקס באופן חופשי ללא כוחות, או שיש כוח חיכוך מסתורי שמאט את תנועתו בציר זה על אף שמדובר בוואקום?

  48. לאריאל
    אם תראה לי בעייה שרק בעזרת מסה ואנרגיה אפלה אפשר לפתור אז אני מתנצל בפני כולם ומזמין אותך לארוחה בכייף. אתה לא מתאר לעצמך איזו מעמסה תוריד מעל כתפי.
    יהודה

  49. אלבנטזו,
    תודה על התשובות. אני אמצא את ההרצאה של ג׳יימס גייטס בנושא. היא היתה קצת יותר מפורטת. בקשר לשון קארול, אני צריך טיפה לחשוב על מה מפריע לי (אם הבנתי נכון הוא מסביר מדוע לא יווצרו מוחות בולצמן אבל מאפשר היווצרות יקום שלם?)

  50. יהודה,
    מעבר לכל מה שכל האחרים כותבים אני חייב לציין שבחיים שלי לא שמעתי על חישוב יכול להניב שתי מהירויות שונות. מה זה, תוכנית כבקשתך? בחיים שלי לא שמעתי על אמונה כל כך חזקה בחלקיקים שטסים קרוב למהירות האור משני הצדדים, שאנחנו לא מגלים, תוך סלידה אינסופית למה שהפיזיקאים קוראים חומר אפל. אתה באמת לא קולט שאתה מחליף חומר אפל שהפיזיקאים טוענים שיכול להסביר את ה bullet cluster בחומר אפל אחר שלפי ויקי (שהיא, מה לעשות, יותר מהיימנה ממך) אמור להיות בעל מהירות הרבה יותר גבוהה מהאור ואילו אצלך קרובה למהירות האור משני הצדדים (תלוי כמה מקשים עליך) ושבכל מקרה, אם מהיר מהאור הרי שזה סותר לחלוטין את היחסות ולא יכול להסביר את ה bullet cluster כי מה שקורה שם הוא שרוב שהגרוויטציה לא נמצאת איפה שרוב החומר המוכר נמצא. איך החלקיקים שלך יכולים להסביר זאת? הפרש לחצים ממש לא יכול להסביר זאת. הפרש לחצים זו סתם מנטרה.
    אתה לא קולט שתורת השדות והמודל הסטנדרטי שמים גבול אנרגטי לחלקיקים שמבצעים אינטרקציה עם החומר הידוע וכל קנדידט חדש לא יכול לבצע את האינטרקציה שאתה מבקש מהחומר האפל שלך לבצע?

  51. יהודה,
    מעבר לכל מה שכל האחרים כותבים אני חייב לציין שבחיים שלי לא שמעתי על חישוב יכול להניב שתי מהירויות שונות. מה זה, תוכנית כבקשתך? בחיים שלי לא שמעתי על אמונה כל כך חזקה בחלקיקים שטסים קרוב למהירות האור משני הצדדים, שאנחנו לא מגלים, תוך סלידה אינסופית למה שהפיזיקאים קוראים חומר אפל. אתה באמת לא קולט שאתה מחליף חומר אפל שהפיזיקאים טוענים שיכול להסביר את ה bullet cluster בחומר אפל אחר שלפי ויקי (שהיא, מה לעשות, יותר מהיימנה ממך) אמור להיות בעל מהירות הרבה יותר גבוהה מהאור ואילו אצלך קרובה למהירות האור משני הצדדים (תלוי כמה מקשים עליך) ושבכל מקרה, אם מהיר מהאור הרי שזבהסותר לחלוטין את היחסות ולא יכול להסביר את ה bullet cluster כי מה שקורה שם הוא שרוב שהגרוויטציה לא נמצאת איפה שרוב החומר המוכר נמצא.
    אתה לא קולט שתורת השדות והמודל הסטנדרטי שמים גבול אנרגטי לחלקיקים שמבצעים אינטרקציה עם החומר הידוע וכל קנדידט חדש לא יכול לבצע את האינטרקציה שאתה מבקש מהחומר האפל שלך לבצע?

  52. יהודה
    לא הבנתי 🙂 אתה אומר לי – “התצפית שלך לא מתאימה לתיאוריה שלי, ולכן אני בוחר להתעלם ממנה”.

    נשמע לי קצת כמו הדת….

  53. יהודה אם אני אציג בפניך בעיה שטענת שהתאוריה שלך פותרת אותה ללא צורך במסה/אנרגיה אפלה אתה תזנח את התאוריה הנוכחית שלך?

  54. לניסים
    אתחיל עם אנדרומדה. אל תהיה כל כך בטוח שנתנגש בה . אצלי אין גרביטציה והיקום הגזי עם סיבובן של הגלקסיות ייצור כידוע אפקט מגנוס והגלקסיות עלולות לאכזב את ניוטון. כי מה הסיכוי ששני כדורי רגל מסובבים יפגשו?. אפסי!.
    ולשאלתך הראשונה. כשאיינשטיין כתב את התיאוריה שלו הוא הוסיף קבוע קוסמולוגי שאחר כך הוא התנצל ובסופו של דבר הוא צדק. מה שצריך לעשות זה תיקון זעיר כדי שיתאים לתיקון הזעיר שלי. תיראה, גם בברית מילה עושים לפעמים תיקונים.
    האמת לא טיפלתי ב”שעשוע” הזה -התכוונתי ל: “פושינג גרביטי יחסותי” (כל הזכויות שמורות על השם) ואולי כדאי לראות באמת מה מהות השינוי שיש לעשות .
    יאללה, הלך עוד סוף שבוע!
    יהודה.

  55. יהודה
    ועוד משהו קטן. אתה אומר, להבנתי, שנוסחת הכבידה שגוייה במרחקים של עשרות, או מאות, שנות אור.
    אז,, למה אנחנו הולכים להתנגש באנדרומדה, עוד לפני שיגמרו את הרכבת הקלה בתל-אביב?

  56. יהודה
    תורת היחסות הכללית מבטאת את הכבידה בעזרת נוסחה “פשוטה” שנותנת תוצאה זהה למשוואה של ניוטון. אני מבין שטתה מציע שינוי של המתמטיקה במקום חומר אפל. קשה לי לקבל שבגלל שלא מצאנו במאיצים יתוג חלקיקי של אותו חומר אפל, שהמתמטיקה של איינשטיין שגויה.

    ניוטון טעה, כי הנחות הבסיס שלו לא היו מדוייקות. איזו הנחה בסיסית של איינשטיין שגויה, לדעתך?

  57. תודה לך ישראל על הזכרת דנידין. מכאן ואילך אשמח להשתמש בכינוי החביב הזה במקום המונח הקודר “אפל”.
    חלקיקי דנידין והרווחים ביניהם הם האחראים לתופעות הפיסיקה השונות, וההבדלים בין התופעות השונות נקבעים על פי היחס בין הדנידין והרווחים, דהינו הצפיפות.
    הסיבה לזה שלא מצאנו דנידין בסביבתנו הקרובה היא בדיוק בגלל זה שהוא בלתי נראה. וכפי שאת דנידין של שרגא גפני (און שריג) אפשר לגלות בתחבולות, גם כדי לחשוף את דנידין שלנו נחוצות לנו תחבולות שאני סומך על הגניוס האנושי שיצליח למצוא כאלה.

  58. תגיד יהודה, מי זה הגאון המוכשר שצילם את ההרצאה שלך ביוטיוב? איכות הצילום פשוט זוועה, החל מהדקה השלישית בקושי רואים צלליות, וגם איכות הקול ממש על הפנים.

    איך בן אדם מצליח להגיע לתוצאה איכותית כל כך ?

    לפחות גערת בו ?

  59. לנקודה צפה ולישראל
    החיכוך קיים עוד מזמן היווצרות מערכת השמש והוא קיים גם כיום גם בלי להסכים עם הפושינג גרביטי, ובמערכת השמש וכוכבי הלכת התגברו עליו, לפחות חלק מכוכביי הלכת. כל מי שנע במסלול מתאים של מקום ומהירות המשיך להסתובב סביב השמש. העליתם בעיה של כיוון כך שחייב להיות וקטור של כוח בכיוון התנועה. אולי זה נכון לא התעמקתי בכך. פתרון אפשרי הוא אולי הכוח הצנטרפוגלי של כל הגלקסיה או אולי שגם החלקיקים עצמם מסתובבים מסביב לשמש כמו כל חומר אחר, כלומר אין סיבה למה שהפושינג גרביטי לא יפעל גם על החלקיקים עצמם, זה שהם קטנים לא אמור להשפיע, הגודל לא קובע. ואולי דבר אחר?. מה שכן הרי מערכת השמש ידעה להתגבר על חיכוך תמיד ואין סיבהשלא תעשה זאת גם כיום עפ הפושינג גרביטי.
    בכלל, שימו לב שכוכביי הלכת והירחים, רובם ככולם, סובבים באותו כיוון אז יש סיבה לכך.
    לא הייתי מייחס לזה יותר מידי משמעות לבעיית החיכוך, אבל זה חומר למחשבה וזה יעסיק אותי.
    נא להיות אופטימיים ושזה לא יגרום לחיכוך בין המגיבים.
    אז נקודה צפה וישראל אני מקווה שעניתי לפחות חלקית או זמנית לבעיית החיכוך.
    יום טוב ובהיר
    יהודה

  60. לאבי בליזובסקי
    תגובה שלי ממתינה להחלטתך . למען הסדר הטוב, הייתי מציע שתיתן רשימה של כללים שיקלו עליך ועלינו . אמור מה אתה מצפה מתגובה. למשל אורכה אולי וכדומה. לא את כל הכללים כי אני בטוח שיש לך כמה סודיים. דבר נוסף אולי רצוי שתבחר שמות של מגיבים שבהם תיתן אמון עד שהם “ילכלכו” פעם.
    דעתי זאת משותפת גם למגיבים אחרים
    יום טוב אבי
    והמשך עבודה ברוכה
    בהערכה!
    יהודה

  61. להלן דעתי לגבי התגובות האחרונות

    ניסים
    אני מקבל את תורת הייחסות אבל היא זקוקה לשידרוג במרחקים.

    ליאצי
    במערכת שמש באנדרומדה הגרביטציה היא כמו במערכת השמש שלנו, אבל המשיכה בין שתי מערכות השמש היא שונה משלנו. זאת הכוונה למושג “במרחקים גדולים”. כלומר, אין הסכמה שאם המרחק בין שני כוכבים הוא אלפי שנות אור הגרביטציה ביניהם תהיה על פי נוסחת ניוטון.
    לאריאל
    אם היקום מלא חלקיקים הנעים מכל מקום לכל מקום, המתנגשים ביניהם אזי הם מגדירים גז, ולכן, יש להם את כל התכונות של גז כמו: נפח, חלקיקים למטר מעוקב, וגם לחץ וגם הפרשיי לחצים וגם רוחות שפועלות מיליוני שנים וקובעות עובדות בשטח.
    שמוליק
    אמרתי מבחינה ניוטונית הם נעים פי 1.3 ממהירות האור , אבל מיד הוספתי שמבחינה יחסותית הם נעים קרוב למהירות האור. תבין שיש עם הכרזה כזאת בעיה, כי אני לוקח מסקנות מתורה מדעית קיימת ומקבל אותן ברעיון מדעי שאני מפתח וזה לא בהכרח חייב להיות כך. לכן הפרדתי ואמרתי את שתי האפשרויות.
    אריאל
    אני לא שיניתי את מסקנותיי על פי אלבנזו, אז תקן את עצמך בבקשה. רעיון הפרש הלחצים קיים אצלי שנים בלי שום קשר לאלבנזו.
    ולגבי ההסבר האלגנטי של המסה האפלה , לא כל אחד יסכים עם זה. למשל חמורים לא יסכימו לאכול רק תבן אפל אפילו אם תגיד להם שהוא ממלא את כל היקום והוא מזין בצורה אלגנטית כמו תבן בריוני.
    לישראל
    החומר האפל אמור להמצא מחישובים גרביטציונים בעיקר בקצה הגלקסיה או בענני הגזים שמחוץ לה. ליד כדור הארץ הוא “דליל”.
    לנקודה צפה
    אתה מדבר על סבלנות??, את החומר האפל מחפשים כבר שמונים שנה, בלי הצלחה יתרה, אין לי בעיה אם אתה רוצה שנמתין בסבלנות עוד כמה שעות!
    יום טוב לכולם
    סבדרמיש יהודה

  62. שמוליק,

    בנוגע לשני הדברים.

    1. נתחיל מהסרטון. הרושם הראשון הוא, כמובן, שניל דגרס-טייסון (אף על תרומתו האדירה למדע פופולרי והבאת הפיזיקה אל אנשים סקרנים שאינם בהכרח בעלי הידע המתמטי הנחוץ לקרוא מאמרים), פשוט הופך את כל הסיפור לבדיחה אחת גדולה ושם את הצורך שלו לבדר הרבה, הרבה לפני הפאנל. אני לא סתם מלכלך עליו, יש לזה רלוונטיות גדולה למה שאני הולך לומר: למרות שאורך הסרטון 12 דקות, הנושא ששאלת אותי לגביו בקושי תופס חצי דקה. יש מעט מאוד מידע, ולכן קשה לענות. אבל באופן עקרוני – רעש וצלצולים. טייסון מאוד נהנה לשחק אל הקהל ולהדהם, אבל בסופו של דבר ראית שאף אחד לא מתרגש יותר מדי, והסיבה היא פשוטה. יש המון משוואות שניתן לכתוב, ובין רבות מהן יש קשרים (טרנספורמציות) שמעבירות פתרונות של אחת לשניה. עצם העובדה שאלגוריתם מסוים (יש הרבה כאלה) שהופך משוואה למשהו ויזואלי נותן תוצאות דומות בשני מקרים, לא אומר שיש קשר סיבתי עמוק בין הפיזיקה שמאחורי המשוואות המדוברות (שים לב גם שהוא טוען רק שהגרפיקה של המשוואות דומה, טייסון הוא זה שהופך טענה זו ל”אותן משוואות”). יתכן ששתי המשוואות מקיימות סימטריה מסוימת שמקשרת ביניהן, אבל זה לא אומר שהיקום הוא קוד. בנוסף, יש הבדל ענק בהתאם לאילו משוואות הוא מציג. למשל, אם הוא מציג משוואות תנועה (שאומרות לנו איך חלקיקי פיזיקלי צריך לנוע) או משוואה שמתארת את אנטרופיית השזירה בתורה סופרסימטרית. במשוואה השניה יש הרבה גורם אנושי – הרבה דברים שאנו הגדרנו (למעשה, כל מושג האנטרופיה הוא מלאכותי. מאוד שימושי, אך מלאכותי, כמו פונ’ חלוקה למשל). במקרה זה יתכן שהקשר בין המשוואות הוא בכלל תוצאה של הגדרות שלנו. לבסוף, צריך להעיר על הצד של error correction: אני מניח שהמשוואות שהוא מדבר עליהן הן משוואות אופטימיזציה של אלגוריתם כזה או אחר. דמיון יכול לנבוע מכך שמדובר באופטימיזציה, או מבחירת האלגוריתם. לסיכום, דמיון בין משוואות (או בין הצגה גרפית שלהם) הוא לא בעל משמעות כלשהי א-פריורי בפיזיקה. יתכנו לכך מליון ואחת סיבות, ורובן נסיבתיות בלבד.

    בנוגע לשון קארול, לא קראתי את המאמר, אבל כבר מקריאת הבלוג ברור שיש כאן כמה דברים שצריך לשים לב אליהם. ראשית, יש כאן המון סמנטיקה. שים לב שהוא אומר במפורש שהמודלים של הפרעות קטנות באינפלציה מדויקות גם לשיטתו, הוא פשוט לא קורא להן פלקטואציות של הוואקום הקוונטי, אלא דהקוהרנטיות שנובעת מהיקומים המרובים (וההתרחבות בהן ושינוי הטמפ’) שגורמת הלכה למעשה למדידה שאופיה סטטיסטי. מבחינה מתמטית אין כאן הבדל, וזה יהיה נכון גם (להבנתי, יש מקום לקרוא את המאמר המלא כמובן) גם אם נחבר באופן קונפורמי יקום ענק ליקום זעיר, מה שפנרוז מדבר עליו. יש לי הרגשה שזה גם לא ישנה לתמונה של לורנס קראוס או לתמונות אחרות של emergent universe, אבל שוב – צריך לקרוא ולא היה לי זמן לזה עד עכשיו.

    בנוסף, גם ההבדלים שקארול כן טועה שיתקיימו, נובעים באופן ישיר מהנחה שלו על ההתפתחות של מצב קוונטי בזמן, שנובע מתמיכתו בפרשנות מסוימת של מכניקת הקוונטים (ספציפית, מקרה מסוים של עולמות מרובים). מה שאומר שאם הוא צודק, זה נחמד מאוד למצוא הבדלים תיאוריים בין אינטרפטציות, אבל לא ברור איך ניתן לבנות מודל כזה בלי לדעת אם האינטרפטציה נכונה. כלומר, יש כאן מעגל כלשהו שלי לא ברור איך הוא נשבר (כרגע, אני לא טוען שלא ניתן לשבור אותו).

    מצטער שלא עזרתי יותר, אבל כדי לתת תשובות טובות יותר צריך ללמוד את הנושא (וזה נכון תמיד, והלוואי שזה היה ברור לכולם כמו שזה ברור לרוב המוחץ של האנשים). אשתדל השבוע להתעמק לפחות בנושא של קארול (שהוא הרבה יותר מעניין בעיני) ואולי אוכל להרחיב. בינתיים אם יש לך שאלות ספציפיות, אתה יכול לנסות לשאול למרות שסביר שאענה שכדי לענות אני צריך לקרוא את המאמרים ולעבור על המתמטיקה.

  63. באמת ישראל, את בוסון היגס מחפשים 40 שנה, ועדיין לא מתחייבים ב 100% שמצאו. את הניוטרון חיפשו 20 שנה.. אתה בטוח ש”אי אפשר למצוא קצת”? טיפה סבלנות 🙂

  64. יובל

    אם לדבריך החומר האפל נמצא בתוך מערכת השמש והוא מהווה אחוז כה ניכר מסך החומר ביקום, מדוע אי אפשר למצוא קצת כאן בארץ או בדרך לירח לדוגמה?

    קשה להתווכח עם דנידין בלתי נראה שעולה על משקל המראה 70 קילו.

  65. יהודה, בתחילה טענת שהבעיה שיש לנו היא שבמרחקים גדולים הכבידה כפי שאנחנו מתארים אותה (ריבוע המרחק) לא נכונה. לאחר שאלבנצו הראה לך ששינוי המעריך יגרום לעוד בעיות, הוספת למשוואה גם הפרש לחצים ופושינג גרביטי(למרות שזה בניגוד להסתייגות שלך למסה אפלה) ואתה עדיין נתקל בבעיות(שוב תודה רבה לאלבנצו). תזכיר לי איך זה יותר אלגנטי או הגיוני יותר מהוספת של מסה אפלה לחישובים שלנו?

  66. לאחר שדיברנו על חלקיקים אפלים מהירים מהאור פי 1.3(!) שהם אינם החומר האפל(!), למדנו שהחומר האפל ליד גוף מסיבי הוא זה שאחראי על עיקום קרני האור ולא הגוף המסיבי והרווחים בין החלקיקים הם הם התווך הנושא של הגלים האלקטרומגנטיים, נדבר על אלכימיה ושימושיה השונים בלהתגבר על החוק השני של התרמודינמיקה.
    יהיה מעניין

  67. אין לנו סיבה להניח שבגלקסיות אחרות שרירות פיסיקות אחרות, אך היות שהגלקסיות נבדלות אלה מאלה בגודל ובמסה, מותר לנו להניח כי לכל גלקסיה קבועים פיסיקלים הייחודיים לה.

  68. יאצי, להבנתך כי כוח הכבידה הוא תוצר של גוף/עצם:
    קיומו של גוף הוא תוצר של צפיפות מקומית של החומר האפל, וכך גם קיומו של כוח הכבידה. דהינו, לא קיומו של גוף יוצר גראביטציה אלא קיומה של גראביטציה וקיומו של גוף נוצרים שניהם במקביל מגורם שלישי – הלא הוא החומר האפל.

  69. יאצי
    כדור הארץ מושך את הירח באותו כוח שהירח מושך את כדור הארץ. שניהם מסתובבים אחד סביב השני, כשציר הסיבוב נמצא במרכז הכובד המשותף. וזה יותר מסובך מזה, כי התנועה היא אליפטית.

    כולם מסכימים שבכל מקום, במרחקים קצרים, חוק הכבידה הוא החוק של ניוטון, כלומר – הכוח תלוי במסות ובריבוע המרחק.

    השאלה היא האם החוק הזה תקף גם במרחקים גדולים. השאלה נשאלת – כי אנחנו רואים תופעות מוזרות במרחקים גדולים. ההסבר המקובל הוא שקיים חומר נוסף שאנחנו לא רואים ישירות – ולחומר הזה יש מסה. יש עוד תופעות שהחומר הזה מסביר, ואנחנו יודעים הרבה תכונות שלו.

    הסבר נוסף, הוא מה שיהודה אומר – חוק הכבידה של ניוטון אינו תקף במרחקים גדולים.

    אני לא פיסיקאי, אבל כש-99.9% מהפיסיקאים אומרים משהו, אני מהמר שהם צודקים. וזה במיוחד נכון שההסבר המתחרה נראה לי לא הגיוני.

  70. יאצי
    “מכיוון שכדה’א יותר מסיבי אזי הוא מושך אליו את הירח ולא להיפך”.

    גם להיפך, אבל רק בשישית מהכוח שהארץ מושכת את הירח אליה.

    “האם הכוונה שבאנדרומדה הגרביטציה פועלת אחרת או בכלל לא קיימת או משהו אחר?”

    לא נראה לי שזה מה שיהודה טוען. אך אם אנו מתייחסים לכוח המשיכה ככוח דחיפה – שזה מה שטוענת תיאוריית הפושינג גרביטי – זה יכול בהגיון מסויים לשפוך אור על אנמוליית הסיבוב של הכוכבים בשולי הגלקסיות, שהיא הטריגר לתאוריית החומר האפל, למרות שקיימות עוד סיבות המצדיקות את קיומו.

    הבעיה העיקרית שיודה שלנו קובל עליה היא שעדיין אי אפשר להיכנס לסופר ולקנות 2 קילו חומר אפל וסלסלה של גרביטונים.

  71. יהודה, ישראל, יובל
    לפי הבנתי כוח הכבידה זה תוצר של גוף/עצם. ככל שהגוף יותר מסיבי כך הגוף מייצר יותר כוח שמושך אליו גופים אחרים (בעלי מסה יותר חלשה ממנו).
    בין כדור הארץ לירח פועל כוח משיכה שמורכב מהמסה של כדה’א והירח. מכיוון שכדה’א יותר מסיבי אזי הוא מושך אליו את הירח ולא להיפך. כמובן שגם גרמי שמים אחרים משתתפים בתהליך, כמו השמש למשל.
    כלומר לא משנה אם זה כאן או בגלקסיית אנדרומדה, בין הגופים מתקיים כוח משיכה שנובע מתוך ובגלל הגופים עצמם.
    להבנתי, גם לחלקיק יסודי יש כוח שמושך אליו חלקיק אחר.
    מה שאני לא מצליח להבין זה למה אתם מתכוונים כאשר אתם אומרים “במרחקים גדולים”.
    האם הכוונה היא לתווך שנמצא בין גוף אחד לגוף משני? האם הכוונה שבאנדרומדה הגרביטציה פועלת אחרת או בכלל לא קיימת או משהו אחר?

  72. יהודה
    זה לא מסתדר לי. בתורת היחסות הכללית, להבנתי, כוח הכבידה לא נגמר בשום מרחק. עזוב כרגע תצפיות שמראות שיש עוד תופעות – תורת היחסות אומרת משהו מאד מסויים.

    או שאתה מקבל את תורת היחסות או שלא.

  73. מה קורה לכם? 🙂
    בתנועה מעגלית יש כוח כח – כוח צנטריפטלי. במקרה שלנו כוח הכבידה הוא הכוח הצנטריפטלי.
    כוח צנטריפוגלי הוא כוח מדומה, שכחתם?

  74. ישראל, לתהייתך “אנחנו רק מנסים להבין איך זה שהארץ מושכת את הירח אם אין כלום באמצע, ופושינג גרביטי נותן תיאור יפה וברור. אך קיימת בעיית החיכוך המרכזית, ועוד שלל בעיות משנה”, שני דברים:
    א) זה שטרם מצאנו שיש משהו באמצע אינו אומר שבאמת אין. קיומו של החומר האפל במרחב התוך-גלקטי הוא כבר עובדה מוכחת תצפיתית, ואין שום סיבה להניח כי הוא לא נמצא גם בתוך מערכת השמש ובכל מרחבי היקום בכלל.
    ב) תופעת כוחות משיכה ודחיה, כמו גם תופעת החיכוך, מוגדרות מתמטית בפיסיקה הידועה. ניסיון להגדיר באמצעותן את עצמן הוא מעגלי ומיותר.

  75. יהודה, כתבת: “יש שלושה כוחות שפועלים כאן גרביטציה חיכוך וצנטריפוגאלי והתנועה תהיה הווקטור השקול של כל הכוחות האלה. לא רואה בעיה.” אם הוקטור השקול לא מצביע בכיוון מרכז המעגל, בהחלט יש בעיה, כי המסלול המעגלי לא ישמר. ויש לך כח שובב שקוראים לו חיכוך והוא בניצב.

  76. ישראל הקדים אותי בעניין החיכוך, אבל יהודה, אני חייב לחדד משהו..

    כתבת “הכוח הצנטרפוגלי של כוכב הלכת למשל יהיה שווה לגרביטציה הנוצרת פחות החיכוך.” האם שכחת מה זה חיכוך? כח צנטרפוגלי הוא בניצב לכיוון התנועה, החיכוך הוא עם כיווון התנועה! אי אפשר לחסר חיכוך מגרביטציה אם הם בניצב זה לזה.

    ישראל צודק – כדי להתגבר על החיכוך חייבת להיות רוח נוספת עם כיוון התנועה וכמו שהוא כתב, זה יוצר מצב אבסורדי.

  77. לישראל
    לא מסכים אתך. ספינות מפרש מסוגלות לנוע גם נגד כיוון הרוח.
    יש שלושה כוחות שפועלים כאן גרביטציה חיכוך וצנטריפוגאלי והתנועה תהיה הווקטור השקול של כל הכוחות האלה. לא רואה בעיה.
    חייבים כניראה לעשות ניסוי
    טוב הגיע זמן לישון
    לילה טוב
    יהודה

  78. יהודה

    ספינה ששטה בתנועה מעגלית, חייבת מקור אנרגיה כדי להתגבר על החיכוך עם המים.

    אם לטענתך ה”רוח” היא מקור האנרגיה, היא חייבת להיות בדיוק בכיוון תנועת הספינה, מה שבלתי אפשרי כי כל ספינה אחרת, כולל אילו ששטות בכיוון ההפוך, יצטרכו רוח בכיוון שלהן.

    עניין נוסף הוא שחיכוך מוליד חום, גם בדוגמת הספינה המפליגה שלך, וחום הוא אנרגיה שצריכה להגיע מאיפה שהוא.

    בדוגמת הספינה שהבאת הרוח היא מקור האנרגיה. הדבר לא יכול להתקיים במקרה של חלקיקי לסאז’, כי שכפי שציינתי, גם תנועה מעגלית בכיוון ההפוך תצרוך אנרגיה מאותם החלקיקים.

    יובל

    מאיפה באה פיזיקה לעולם – שאלה נכבדה לבני תמותה פשוטים כמונו.

    אנחנו רק מנסים להבין איך זה שהארץ מושכת את הירח אם אין כלום באמצע, ופושינג גרביטי נותן תיאור יפה וברור. אך קיימת בעיית החיכוך המרכזית, ועוד שלל בעיות משנה.

  79. ישראל שפירא
    לפי ההיגיון הזה ספינות מפרש לא יוכלו לשוט בים כי יש להם חיכוך עם המים והאוויר???. הסיבה שהן בכל זאת לא מפסיקות לשוט כי הרוח כל הזמן נושבת ומתגברת על החיכוך לכן גם בפושינג גרביטי, כל עוד החלקיקים ממשיכים להגיע למערכת הם ימשיכו ליצור גרביטציה שתצליח להתגבר על החיכוך. מקווה שלא אצטרך להסביר זאת שוב.
    לאריאל
    לשאלתך הראשונה
    יש דרכים לא קלות אבל אפשריות להוכיח את הרעיונות שטמונים בפושינג גרביטי או לפחות לגבי אי נכונות ריבוע המרחק של הגרביטציה במרחקים גדולים. למשל המערכת הכפול כפול אפסילון לירה אמורה להסתובב לאט יותר מאשר על פי ניוטון ועוד כמה אפשרויות
    ולגבי השאלה השנייה
    הפרש לחצים שהגיונית קיים ביקום העמוס בחלקיקים נעים יכול ליצור תנועה מבלי להצטרך לגרביטציה של מסה. הפרש לחצים יכול להסביר את התפשטות היקום המואצת ללא צורך במסה אפלה אבל זה להרצאה שלמה.
    לילה טוב
    יהודה

  80. דיבורים על “חיכוך” או “ספיגה” של “חלקיקי גז” הם ניסיון להגדיר פיסיקה באמצעות פיסיקה. זו הגדרה מעגלית אשר תמיד תשאיר טעם של “עוד” ולא תפתור את השאלה כיצד באה הפיסיקה לעולם.

    בשעתו היה לי ויכוח עם יואב ליטבק שהביא מודל ציורי יפה המתיימר להסביר תופעות פיסיקליות יסודיות באמצעות תופעות פיסיקליות מורכבות. הויכוח בינינו הסתיים בניתוק יחסים, שעליו אני מצטער. אך הנקודה שעליה אני מתעקש היא כי חוקי הפיסיקה הידועים היום הינם תולדה של חוקי פיסיקה בסיסיים שעדיין בלתי ידועים לנו.
    על דבר אחד אני מוכן להסכים, והוא כי גם חוקי הפיסיקה הבסיסיים שעדיין אינם ידועים חייבים לענות לנוסחאות כימות מתמטיים כלשהן.

  81. תורת היחסות הכללית היא מודל המכמת תופעות פיסיקליות אך אינו מסביר אותן.
    “התעקמות” האור במעברו ליד גוף מסיבי מוסברת על ידי מפל צפיפויות של החומר האפל. צפיפות גדולה מתבטאת בגופים שמכירה הפיסיקה העכשווית; צפיפות נמוכה מתבטאת במה שהפיסיקה היום מכירה כ”חלל ריק”. חלקיקי החומר האפל בונים גופים פיסיקלים; הרווחים שבין החלקיקים הם התווך הנושא את הגלים האלקטרומגנטים.

  82. יהודה

    גוף שנע באוויר, כולל בתנועה מעגלית, יאט ויעצר אחרי פרק זמן ממושך.

    כפי שציין פיינמן בהרצאה שהבאת, כדוה”א היה צריך להאט ולהעצר ב4 מיליארד השנים שהוא מקיף את השמש. זה לא קורה, הוא אפילו לא מאט.

  83. ניסים
    תורת היחסות כן מגבילה את הטווח של הכבידה אבל היא עושה זאת עם האנרגיה האפלה שבטווחים גדולים דומיננטית יותר מהכבידה וכיוונה הפוך.
    נסה דוגמא אחרת.
    ערב טוב ניסים

  84. יהודה יש לי שתי שאלות בשבילך:
    1) האם יש דרך לבדוק את נכונות התאוריה שלך?( אולי ניבוי נחמד בצד או הסבר לתופעה שעדיין אין לנו הסבר בשבילה )
    2) אתה טענת שהוספת מסה אפלה למודל הנוכחי שלנו לתנועה של גלקסיות היא דרך חמקמקה כדי להתאים את התאוריה שלנו לתוצאות הניסויים. איך הוספת מודלים נוספים(פושינג גרביטי והפרש לחצים) עוקפת את הבעיה שתיארת?(כפי שזה נראה רק סיבכת את העניינים)

  85. ניסים
    אולי זה יפליא אותך אבל אני אסביר זאת עם תורת היחסות הכללית. אני לא מתנגד לה. אם תהיה בעיה שלא אוכל להסביר עם הפרשי לחצים עם יחסות ועם הפושינג גרביטי ואהיה חייב תוספת של מסה אפלה אזי אני בשמחה ארים ידיים ואתנצל בפני כולם. בינתיים אין צורך במסה אפלה. תביא דוגמא מעשית ואז ניראה אם נתגבר עליה.
    ערב טוב
    יהודה

  86. לישראל שפירא
    מי אמר שאין חיכוך במערכת פושינג גרביטי ?, יש חיכוך אך הוא לא צריך להפריע לגרביטציה פשוט הכוח הצנטרפוגלי של כוכב הלכת למשל יהיה שווה לגרביטציה הנוצרת פחות החיכוך.
    ניסים
    למה לאנשים קשה להבין שהמסה האפלה היא היא שינוי המדידות כדי שיתאימו לנוסחת הגרביטציה גם במרחקים גדולים של הגלקסיות??
    איתן
    אני מאמין שאין גרביטציה במרחקים הקוסמולוגיים הגדולים ואם מזה נובעת סתירה אז מישהו צריך לסגת עם התיאוריה שלו.
    ערב טוב
    סבדרמיש יהודה

  87. יהודה
    ככל שידוע לי, ההנחה של תאוריית היחסות היא שכבידה אינה משתנה ביחס למרחק.
    כנ”ל לגבי תאוריית הקוונטים.
    אחת רואה את כוח המשיכה כתוצא של עקמומיות המרחב, השנייה רואה אותו ככוח הנישא בחלקיקים תיאורטיים אולם שתיהן לא מתייחסות לשינויים במשיכה במרחקים גדולים (מלבד הירידה בהשפעה יחסית ביחס למרחק בין שני גופים)
    אם אתה טוען שכוח המשיכה שונה ממה שמקובל, או שקיים “פושינג גרוויוטי” (אני מודה שלא הבנתי אם אתה רואה זאת ככוח נוסף או כמופע אחר של כוח המשיכה), אזי אתה טוען כנגד שתי התאוריות הללו ומן הראוי שתספק הסברים לכך.

  88. יהודה
    אתה יכול להתנגד להיפותיזה של חומר אפל, אבל לטעון שמישהו מנסה לשנות מדידות זו האשמה די חמורה. אתה טוען שיש מאמר שמישהו זייף?

  89. לאיתן
    לא אני העברתי את הנושא לפסים אישיים.
    אני לא מתחרה בתורת היחסות ובתורת הקוונטים. מאיפה ראית את זה?
    ס”ה אני מתנגד למסה והאנרגיה האפלים ורואה בהם ניסיון לשנות את המדידות המתקבלות בשטח כדי שיתאימו לנוסחה.
    אתה גם טוען שאני יוצא נגד תיאוריה מבוססת. אפשר לדעת על איזה תיאוריה אתה מדבר? על המסה והאנרגיה האפלות??, מצטער, דעתי אינה כדעתך.
    ולגבי ה”בור” ושאר כינויים, אני חושב שההוא הגזים.
    ולגבי הבעת דיעה במערכת אנונימית , אז הוא אנונימי, אני לא!
    יום טוב
    סבדרמיש יהודה

  90. יהודה
    אין לי כלים להגיב להשערות שלך. אני לא פיסיקאי והכשרתי בנושא מסתיימת בקורס מבוא לפיסיקה א’ ו-ב’.
    לעומת זאת. אני כן מסוגל לזהות התחמקות מתשובה והעברת נושא הדיון לפסים אישיים.
    אם יש לך תאורייה שמתחרה בתורת השדות הקוונטים ובתורת היחסות, מן הראוי לספק איזשהוא בסיס נוסחאתי לתאורייה, להתייחס לבעיות שקיימות בתיאוריה ולתצפיות שמאמתות או מתנגשות בה.
    כשיוצאים כנגד תיאוריה מבוססת, מן הראוי לספק הסברים מה רצף ההשערות שהביא לבניית התיאוריה ולהתייחס לסיבות שגורמות לקהילה המדעית שלא לקבל תיאוריה זו.
    כמו כן, בור בהקשר מדעי מתייחס להיעדר ידע בתחום מסויים. לא כקללה כפי שמקובל בשפה יומיומית.
    אין בכך עלבון שלא לדבר על עילה לתביעה (גם אם לא היה מדובר בהבעת דיעה במערכת תגובות אנונימיות)

  91. יהודה,

    “לרדת לרמה שלי” זה אומר אשכרה להתייחס עניינית לעובדה שהתיאוריה שאתה מנסה למכור לכולם פשוט מספקת פרדיקציות שבמעבדה רואים שהם לא נכונות?

    אתה בור. ולאיים בתביעות מטופשות לא יעזור לאף אחד ולא ישנה את העובדה שאתה מדבר על דברים שאין לך צל של מושג בהם. אתה לא יודע על מה אתה מדבר, ולכן אתה – באופן מילוני לחלוטין – בור.

    לאיים בתביעות רק מוסיף חטא על פשע. אתה מבין, לא משנה כמה אתה לא אוהב את השפה שלי (לדעתי, הסיבה האמיתית שאתה כל כך מתרגש היא לא השפה שלי, אלא תוכן דברי – אתה יודע שאין לך מה לענות כשחושפים שאתה מדבר שטויות שהופרכו לפני יותר ממאה שנה), אני מעולם לא ניסיתי לסתום לך את הפה. מעולם לא ניסיתי לגרש אותך. מעולם לא איימתי עליך. כל מה שעשיתי, זה לוודא שכל אדם שקורא את ה”הגיגים” שלך יודע שמבחינה מדעית הם ערימה גדולה של בולשיט. אתה מנסה לסתום לי את הפה, וזה מראה עליך דברים שהם הרבה יותר גרועים מסתם בורות.

  92. אלבנטזו,
    ברור. בד”כ זו חוצפה להקפיץ שאלה לראש השרשור אבל בגלל, הממ, “העומס” החלטתי להציף מחדש.

    תודה מראש

  93. לכולם
    לא אענה יותר לאלבנטזו, לא רוצה לרדת לרמה שלו. לטובתו, שיזהר מאוד בדבריו שלא אתבע אותו.
    לכנות אותי “בור” וכדומה עלול שלא לעבור בשתיקה .
    יהודה

  94. יהודה,

    ראשית, אני מבטיח לך שאני איני איתן. איתן אמנם צודק במה שהוא אומר לך, אבל הוא מפספס את העיקר: אתה איש בור (ואין כאן שום עלבון: אתה בעצמך מודה שמעולם לא למדת את נושא הגרביטציה מעבר לנוסחא של כיתה ט’, כלומר אין לך מושג ביחסות כללית, בתיאוריה של חומר אפל, וכו’) שמנסה לכפות את בורותו על המציאות..

    וכמובן, מה יגיד איש בור ופחדן כאשר מציבים מולו הפרכות לשטויות שהוא אומר? “אני לא הולך להתייחס לזה. נמאס לי”. ברור שלא תגיב. כי אתה לא יכול להגיב. כי אתה מתעקש ומתווכח על נכונותה של תיאוריה שכמות ההפרכות לה לא נכנסות בתגובה או בעשר תגובות באתר. ואתה מתעקש עליה בגלל בורותך. אם היית טורח ללמוד את הדברים שאתה מדבר עליהם, לא היית אומר מה שאתה אומר. אתה מבין, במקום לקבל את ההסבר הברור (שהסיבה שכל מי שמבין בגרביטציה דוחה את התיאוריה הזו, היא בגלל שברגע שלומדים את מכלול הראיות והעובדות רואים שהתיאוריה פשוט לא מתארת את המציאות שלנו), אתה פונה להסברים פרנאודיים (“אני היחיד שחשב על זה! פיזיקאים מקצועיים ומצליחים לא יכולים להבין מה שאני אומר! יש קונספציה! יש קנוניה!”), וזאת כי אתה איש בור ומסכן שחי בהכחשה. לראיה, אתה חושב שנסיון בניהול עסק ליעול מפעלים מסמיך אותך לדבר על נושאים מתקדמים בפיזיקה מודרנית. זהו, אין יותר דלוזיה מזה.

  95. למגיב החדש איתן

    בתגובה אחת עשית את הפעולות הבאות:
    1. התחלת בכך שמזלזלים בך-
    תשובה:- מלבד אחד, כולם מגיבים בכבוד . הסכמה לדעתו של האחר או לא, זה לגיטימי .
    2. הצהרת שאתה מתעלם מגורם שיוצר בעיה להשערה שלך ללא הסבר מדוע אתה מתעלם
    תשובה: אחרי שמספר פעמים הסברתי זאת אסביר זאת שוב, לך, איתן. בגרביטציה של מערכת השמש למשל כוח המשיכה שקול לכוח הצנטרפוגלי של כוכב הלכת. אבל כוח המשיכה הוא לא רק הגרביטציה כי אם הוא השקול של גרביטציה, קרינת השמש ו… חיכוך. הרי תמיד מסתובביםחלקיקים שונים במרחב. זה אף פעם לא הפריע לכוכבי הלכת להסתובב ואין סיבה שיפריע אם ההסבר הוא דווקא הפושינג גרביטי.
    3.טענת שאפילו לא תנסה להסביר מכיוון שמגיב אחד פה, לדעתך, לא מסוגל להבין את ההסבר-
    תשובה:- נכון, לדעתי המגיב לא רוצה להבין או לא מסוגל להבין .
    4. אתה מכיר בכך שההשערה שלך איבדה תקיפות בתחילת המאה ה 20 אך לא מתייחס לכך
    תשובה: נהפוך הוא, אני חושב שהגיע הזמן לבדוק כל כמה שנים או חודשים רעיונות כאלה.
    אני בודק גם רעיונות אחרים שמקובלים כבר עשרות שנים, ושאולי יפתיעו אותך למשל שמהירות האור אינה קבועה, שהירח מתרחק רק ב 25 מ”מ לשנה ולא 38 מ”מ, שמשקל גופים משתנה כפונקציה של זמן וכמובן שאין מסה ואנרגיה אפלות.ועוד ועוד. אז מה אם אחרים אומרים אחרת?
    5.האשמת אחרים בכך שהם מאשימים אותך בבעיות שלהם
    תשובה:- כבר אמרו חכחמינו: הפוסל, במומו פוסל. כלומר כשאתה מאשים מישהו במשהו, בדוק קודם כל את עצמך.
    6. ולבסוף, אתה מתייחס לעצמך בגוף שלישי-
    תשובה: לא ראיתי היכן, ואם כן אז מה זה כל כך חשוב?
    אם מישהו היה מציג כך את טיעוניו מולך, האם היה לך קל להתייחס אליהן ברצינות?
    או שאולי היית מבקש ממנו לא לתת לשיקולי אגו להשפיע על אופן הצגתן?
    תשובה-מעולם לא אמרתי דבר ללא מחשבה כשרק שיקולי אגו מנחים אותי.במדע אגו לא קובע. ביעול העסקים בו התעסקתי הייתי ניגש לפועלות הכי רגילות בתהליך הייצור ושואל אותן מה לדעתן הבעייה ותתפלא, העובדות ה”טיפשות” האלה ידעו בדרך כלל מה היא הבעייה והציעו רעיונות טובים לפיתרון. שיקוליי אגו מאוד מאוד מאוד רחוקים ממני.
    אני מקווה שעניתי על שאלותיך
    ומקווה שאתה לא אלבנזו (אם כי מה זה חשוב)
    שבוע טוב
    סבדרמיש יהודה

  96. שמוליק,

    לעיתים אני לא נמצא כמה ימים (או כמה שעות) ופתאום מצטברות המון תגובות, אז אין לי כוח להתעדכן על כל מה שפספסתי (או רצון). בעתיד אחזור אחורה ואסתכל על מה ששאלת, בשאיפה עוד היום (אבל אין הבטחות).

  97. יהודה,

    1. הבנת הנקרא שלך היא פשוט בדיחה. מעורר רחמים שבנאדם מגיע לגיל מבוגר בלי להיות מסוגל להבין טקסט פשוט. אל תדאג, אפרט בדיוק מה לא הבנת (למרות שהמלכוד 22 כאן הוא שכמובן לא תבין מה שאני מסביר גם עכשיו):

    2. בשום שלב לא אמרתי שאתה לא מודע לחיכוך. התייחסתי רק לכך שטענת שהוא לא בעיה. אז מה לעשות, יהודה, הוא בעיה. כשתיאוריה נותנת פרדיקציה ואז אתה הולך למעבדה, מבצע תצפיות, ורואה שהפרדיקציה בפירוש לא מתממשת, זו בעיה. אולי לא בעולם פנטזיה של גימלאים עם ידע בפיזיקה של כיתה ח’, אבל בעולם המדע – בעיה.

    3. מצחיק שאתה כותב “אפילו שריצ’רד פיינמן כתב את זה!” ובכך מייחס לי כאילו נתתי חשיבות מיוחדת לדבריו של פיינמן כאשר במציאות כתבתי שחור על גבי לבן שפיינמן הוא רק דוגמא (שאני משתמש בה כי אתה הבאת אותה), אבל למעשה כל מי שטרח לחקור את הנושא במאה השנים האחרונות יודע שהוא פשוט לא תואם לתצפיות.

    4. אם אתה מתעקש לא להוכיח את דבריך, אלא פשוט לחזור כמו מנטרה על השטות המופרכת (מופרכת במובן המתמטי) שחיכוך הוא לא בעיה, אז זה מראה שאתה חושב שיש פה דמוקרטיה. זה שאתה אומר “אני רשאי ומסוגל לשלא להסכים לכך שהחיכוך הוא בעייה” מצביע על כך שאתה חושב שלכל אחד יש דיעה ולכל דיעה יש ערך, שזה פחות או יותר – דמוקרטיה. מה לעשות, במדע זה לא ככה. אם אתה אומר משהו בלי יכולת לספק לו עדויות אז לדעה שלך אין שום ערך. למקרה שזה לא מספיק ברור אני אגיד זאת שוב – יהודה, לדעה שלך אין שום ערך. וזה לא כי שמך הוא לא פיינמן, זה פשוט כי אתה סתם מדבר שטויות ואין לך יכולת לבסס את השטויות שלך כדרוש בשיטה המדעית: עם מודל מתמטי שמספק פרדיקציות שתואמות את המציאות. מה לעשות, הפרדיקציות של המודל המדובר מנוגדות למציאות לגמרי. הזמנתי אותך בתגובה הקודמת ואני מזמין אותך שוב – הצג בבקשה מודל מתמטי של התורה הנ”ל שאינו מייצר חיכוך (כלומר, לא מנבא פרדיקציות שלא קיימות).

    5. “אני לא אנסה להסביר” = אני פחדן וטיפש מדי, אז אמציא תירוץ שכביכול פוטר אותי מלנמק את דברי. וכן, יהודה, ברור שאני לא יכול להבין. ברור… הרי למה שפיזיקאי מקצועי (ואף אם יורשה לי להעיד, מצטיין בתחומו) שחוקר כבידה כבר שנים יבין משהו טריוויאלי במכניקה קלאסית? ברור שצריך לפחות 40 שנות נסיון בניהול עסק ליעול מפעלים בשביל להבין טיעון קלאסי בנוגע ל”למה חיכוך לא משנה”.

    6. ושוב חוזרים לזה שאתה לא יודע לקרוא ברמה של ילד בכיתה ה’. אתה מאשים אותי בכך שאני לא מבין למה הכוח המכני פרופורציוני למסה, כאשר בתגובה שלי *כתבתי במפורש* את ההסבר לכך. תגיד, אתה דביל או מה? כתבתי במפורש את ההסבר לכך, ומה לעשות – הוא עובד רק במקרים מסוימים (שהם המקרים שהיו מוכרים לפני מאתיים שנה, כשכתבו את התיאוריות האלה). אבל ההסבר הזה לא עובד עבור חלקיקים חסרי מסה או מטען, למרות שאנו רואים בתצפיות שהם כן מושפעים מכבידה (תצפיות, זוכר? הדבר הזה שבעיניך התורה לא צריכה להתאים לו).

    7. מעולם לא זלזלתי בשנקר. כרגיל, המוח שלך בודה שטויות. זלזלתי *בך*. זלזלתי בעובדה שקיים דביל בעולם שחושב שתואר בניהול משנקר ונסיון ביעול מפעלים הופך אותו למוסמך לעסוק בחומר אפל. יש לי חברים טובים שלמדו בשנקר, והם מעצבים מצוינים. אני מזלזל בך בדיוק באותה מידה שאני אזלזל במישהו שלמד ב-אם.איי.טי ביולוגיה ומדעי המוח, וחושב שזה מסמיך אותו לחקור חומר אפל. אני לא מזלזל בביולוגיה או ב-אם.איי.טי, אני מזלזל באדם שחושב שאפשר לחקור נושא X בלי ללמוד את נושא X. המקרה שלך הוא כמובן חמור יותר, כי ה”הסמכה” שלך היא אפילו לא אקדמית. זאת אומרת, לטובת חוקר המוח שהזכרתי מקודם יאמר לפחות שהוא מבין מה זה מחקר, שיש לו נסיון בחקר הטבע, העלאת היפותזות והפרכתן, ביצוע ניסויים, קריאת מאמרים וכו’.

    8. אני לא “אומר” שאתה לא חושב בהגיון. אני מראה בדיוק מה לא הגיוני בבליל השטויות שאתה פולט. בניגוד אליך, אני *בחיים* לא נותן טענה ומסרב לעמוד מאחוריה עם ראיות או הסבר. אני מאשים אותך בטמטום וחוסר הגיון כי אני חושף בכל תגובה שאתה אומר שטויות.

  98. יהודה
    בתגובה אחת עשית את הפעולות הבאות:
    1. התחלת בכך שמזלזלים בך
    2. הצהרת שאתה מתעלם מגורם שיוצר בעיה להשערה שלך ללא הסבר מדוע אתה מתעלם
    3. טענת שאפילו לא תנסה להסביר מכיוון שמגיב אחד פה, לדעתך, לא מסוגל להבין את ההסבר
    4. אתה מכיר בכך שההשערה שלך איבדה תקיפות בתחילת המאה ה 20 אך לא מתייחס לכך
    5.האשמת אחרים בכך שהם מאשימים אותך בבעיות שלהם
    6. ולבסוף, אתה מתייחס לעצמך בגוף שלישי

    אם מישהו היה מציd כך את טיעוניו מולך, האם היה לך קל להתייחס אליהן ברצינות?
    או שאולי היית מבקש ממנו לא לתת לשיקולי אגו להשפיע על אופן הצגתן?

  99. לאלבנזו

    מרגיז לפעמים כמה אדם אחראי כמוך קורא דברים בריפרוף כי הרי ללכלך על יהודה ולזלזל סתם כך במגיבים אפשר גם בלי להתעמק.
    אם היית מתעמק קצת בתגובתי היית רואה שאני מודע לחיכוך אך לא חושב שהוא הבעיה .
    ותתפלא אני רשאי ומסוגל לשלא להסכים לכך שהחיכוך הוא בעייה אפילו שאחד גדול ודגול כמו ריצרד פיינמן אמר את זה! ואין פה עיניין של דמוקרטיה.
    אני לא אנסה להסביר לך למה החיכוך לא חשוב לגרביטציה של הפושינג גרביטי כי אתה לא רוצה להבין וזה בניגוד לקונצפסיה שלך להבין ואולי אתה גם לא מסוגל להבין!
    אתה גם לא תהיה מסוגל להבין גם למה התנגשות החלקיקים דווקא פרופורציונית למסה ולא לפני השטח, כי אתה לא רוצה להבין , וחבל כי על אף הכל יש לך ידע רב וראש בריא על הכתפיים שיכול היה לתרום.
    אתה חושב שאני לא קורא שהתיאוריה כבר הוכחשה לפני שנים??, אבל למדתי בשנקר (זוכר??, הבצפר ההוא שזילזת בו כל כך??) ועוד לפני כן אצל הגננת אסתר בגן הילדים “במבי” שאם לא טיפלת בנושא מסויים מספר שנים אז וודאי שהיגיע הזמן לטפל בו”, אז אתה מספר על מאה שנה שלא חידשו משהו בנושא הפושינג גרביטי??
    אבל אתה כמובן אומר שאני לא חושב בהיגיון שלא רוצה להבין וכו וכו וכו. היי בחור, אתה יודע שהגמל לא מסוגל לראות את הדבשת שלו??

    כל טוב ושיהיה לך ולכל עם ישראל
    שבוע טוב
    נא להגיב בעדינות.
    יהודה

  100. ליובל חלקין
    לא בטוח שיש רשימה כזאת לאבי, גם אני הוותיק ממתין לפעמים בסבלנות לאישור תגובה
    אולי רצוי באמת שתהיה רשימה כזאת!
    רק אחרי שמישהו “ילכלך”, להטיל עליו צנזורה.
    שבוע טוב
    יהודה

  101. 1. לא קראתי את הערך בויקיפדיה, אבל כן ראיתי את ההתייחסות של פיינמן. מה שהוא מדבר עליו שם זה בדיוק התיאוריה המכנית של כבידה של לסאז’ ואחרים.

    2. “פיינמן מבטל אותה בגלל החיכוך דבר שאני לא מסכים איתו”. מה זה פה, דמוקרטיה? מה זה אומר “לא מסכים”? אם פיינמן טועה, תוכיח! (דרך אגב, זה לא פיינמן. כבר חמישים שנה לפניו אנשים הבינו שהרעיון הזה פשוט לא מתאים למציאות בשום צורה). בוא תראה בבקשה מודל של כבידה מכנית כמו שמתואר בסרטון ש*לא* מייצר כוחות חיכוך.

    4. כמובן שחיכוך זו לא הבעיה היחידה, זה פשוט משהו ממש טריוויאלי שאפשר להסביר בשורה אחת ולראות בבירור שהתיאוריה נופלת כי היא לא מתאימה למציאות. יש עוד הרבה אי-התאמות בין הרעיון המדובר והמציאות שלנו. למשל, בתיאוריה המכנית הכבידה פרופורציונות לשטח הפנים של הגוף החווה כבידה ולא למסה. נהוג היה לפתור בעיה זו ע”י כך שאמרו שהחלקיקים בשטף הם מאוד קטנים ולכן למעשה חודרים את הגוף ועושים אינטראקציה עם אבני הבניין, שלהן יש צפיפות קבועה ולכן מקבלים פרופורציה למסה. אבל זה פשוט לא נכון בסקלות מיקרוסקופיות, ובצורה אפילו יותר פשוטה – לא נכון עבור חלקיקים חסרי מסה. יתרה על כך, כיום אנו יודעים שאין דבר כזה “כוח מכני” – כל הכוחות המכניים הם תוצאה של מיצוע על פני אינטראקציות אלקטרומגנטיות. לפיכך, אם אנו מבינים את שטף החלקיקים כמפעיל כוח מכני, הוא לא יעשה אינטראקציה עם חלקיקים יסודיים לא טעונים (מאותה סיבה שלא ניתן לדחוף פוטון). אנו כמובן יודעים באופן מוחלט ומדוד שחלקיקים לא טעונים ו/או לא מאסיביים מרגישים גרביטציה. והרשימה נמשכת ונמשכת…

    אבל מה לעשות, כשמגיע אדם שבאופן מוצהר מסרב ללמוד נושא מסוים לפני שהוא מדבר עליו, ומנסה למכור באתרים פופולריים תיאוריות שהופרכו באופן אמפירי לפני יותר ממאה שנה, קשה לצפות שהוא יקשיב לדברים זניחים כאלה כמו עובדות או הגיון, או שיעשה צעדים מינימליים להבין על מה הוא מדבר, קל וחומר להוכיח את טענותיו (או לפחות לנסות להוכיח אותן. להוכיח אי אפשר, כי הן שגויות).

  102. את התגובות שלי כאן אני מפרסם גם ביקום המקביל של הפייסבוק. אמשיך לעשות כן עד ששמי יוכנס לרשימת הפטורים מהמתנה
    בתודה מראש, אבי

  103. הגרביטציה היא תהליך המתרחש בתוך כל מרחב המאוכלס בחומר אפל ומתבטא בתופעה הנצפית של הימשכות גופים זה אל זה. ניוטון כבר עלה על נוסחה המקשרת בין תכונה של גופים, המכונה מסה, המרחק היחסי ביניהם והתאוצה בה הם מתקרבים זה לזה. להלכה, הנוסחה הזאת טובה ונכונה עבור כל מסה וכל מרחק – אטומים בודדים כמו צבירי גלקסיות, אך בתוך מבנים הקרויים “אטומים” מתגלה סטייה מן הנוסחה, וזו מצריכה הבנה טובה של האופן בו נוצרת התכונה מסה מן החומר האפל.

  104. לכל מאן דיבעיי
    אכן קראתי את תיאוריית לה סאז’ שם מוסבר כיצד הוא רואה את היווצרות הגרביטציה והיא שונה מהדרך שלי. אולי בגלל זה הוא מגיע למסקנה שמהירות החלקיקים חייבת להיות פי 100,000 ממהירות האור.

    הדרך של רבים אחרים ושלי מוסברת בהרצאתו של ריצארד פיינמן באוניברסיטת קורנל החל מהדקה השמינית.
    https://www.youtube.com/watch?v=kd0xTfdt6qw

    • בהמשך, ריצ’רד פיינמן מבטל אותה בגלל החיכוך דבר שאני לא מסכים איתו.
    מי שרוצה שיעיין.
    ויובל חנקין, אם אין לנו בעייה עם טמפרטורת הקפה אז אתה בטח מוזמן. להתראות!
    שבוע טוב
    יהודה

  105. תודה יהודה
    אני אוהב את הקפה שלי קר 🙂
    בעת הציפייה הממושכת להפשרת תגובותיי גיליתי כי להידען יש יקום מקביל בפייסבוק והריני ממליץ לכולנו להעביר את עיתותינו גם שם
    להתראות (Y)

  106. שלום ליובל חייקין
    מה שלומך?, אתה כמובן עדיין מוזמן לקפה אבל קח בחשבון שהוא מתקרר כי היקום כולו מתקרר ובנוסף קבוע הבל והתפשטות היקום (המואצת!) הופכים את הפגישה ביננו לקשה יותר.
    כל טוב
    סבדרמיש יהודה

  107. ישראל שפירא

    יודה, בוא ננסה את זה:
    1. אם המרחק הממוצע של החלקיקים זה מזה גדול והם מתנגשים זה עם זה – נקבל גרביטציה, אבל רק במרחקים גדולים.
    זה סותר את הידוע לנו על גרביטציה גם במרחקים קטנים.

    תשובה: לא!!!, רק במרחקים קטנים תהיה גרביטציה!. ההתנגשויות של החלקיקים בינם לבין עצמם עושות את תנועתם של המתנגשים לאקראית ולכן המידע הכלול בהם במרחקים גדולים הופך לאקראי.

    2. אם המרחק קטן, קיבלנו גז פשוט.
    במקרה כזה לא תהיה כלל גרביטציה, כי לא יווצר הפרש לחצים, כפי שאין גרביטציה בין 2 גופים באוויר.
    תשובה.: שוב אתה טועה. בין שני גופים קרובים (מספר ס”מ) באוויר לא תיווצר גרביטציה כי הדרך החופשית הממוצעת של מולקולות הגז היא מאוד קטנה בסדר גודל של מקרומטרים ולכן החלקיקים מאבדים את המידע על הגרביטציה שהם נושאים בתנועתם המכוונת. אם הגז יהיה דליל יותר והדרך החופשית הממוצעת תהפוך לפחות לגודל עשרות ס”מ אז כן תופיעה המשיכה.

    לגבי סעיף 3 אני לא יכול להסביר לך יותר טוב ולצערי נצטרך להשאיר משהו לפגישה ביננו.
    ניראה לי שאנו מבינים את הפושינג גרביטי בצורה שונה זה מזה

    סיימתי
    הולך לאכול ולנוח. ביי
    יהודה

  108. יהודה,
    המודל הסטנדרטי לא מכיל את החומר האפל. אולי כדאי תקרא עליו קצת.
    תורת השדות מספרת לך באילו אנרגיות ניתן יהיה לאתר חלקיק פוטנציאלי שעדיין לא התגלה. נחש מה? חלקיק כזה שטרם התגלה או בקושי יבצע אינטרקציה או טווח הפעולה שלו יהיה קצר בהרבה מזה של של הכוח הגרעיני החזק. לחלקיק כזה קוראים חומר אפל והוא לא יבצע את מה שאתה רוצה שיבצע

    בנוסף, אתה מאוד נינוח עם חלקיקים שמהירותם גדולה פי 1.3 ממהירות האור. סלח לי, זה פשוט לא רציני

  109. לשמוליק ולניסים

    לי יצאו תוצאות אחרות מאשר בויקיפדיה. מהירותו הניוטונית היא פי 1.3 ממהירת האור ובחישוב יחסותי היא קרובה למהירות האור, ומסת החלקיק היא קרוב לעשר בחזקת מינוס 36 גרם.
    אבל, גם אני אהיה עדין, ואם אתה וניסים מאושרים מהמודל הסטנדרטי שבו אפשר להוסיף תמיד מסה (אפלה) כמה שצריך וכמה שחסר ולקבל כל תוצאה שתידרש אז אני שמח בשמחתכם!
    מה שכן אני אחפש את המאמר בו כתובות מסקנותיך על המהירות והמסה של החלקיק ואז אגיב שוב. בינתיים כל טוב.
    נא להגיב בעדינות
    יהודה

  110. יודה, בוא ננסה את זה:

    1. אם המרחק הממוצע של החלקיקים זה מזה גדול והם מתנגשים זה עם זה – נקבל גרביטציה, אבל רק במרחקים גדולים.

    זה סותר את הידוע לנו על גרביטציה גם במרחקים קטנים.

    2. אם המרחק קטן, קיבלנו גז פשוט.

    במקרה כזה לא תהיה כלל גרביטציה, כי לא יווצר הפרש לחצים, כפי שאין גרביטציה בין 2 גופים באוויר.

    3. אם החלקיקים לא מתנגשים ולא מתאבכים וככלל לא מפריעים זה לזה והמרחק ביניהם קטן – נקבל גרביטציה על פי מודל לסאז’ בכל מרחק.

    אולם אז אנו חוזרים לשאלת פיינמן הבסיסית, שאותה אני שואל אותך כבר שנים: מה עם החיכוך?

    אינך יכול לפטור את השאלה הזו במרחק גדול בין החלקיקים, כי אז אנו חוזרים לסנריו 1, דהיינו שלא תיווצר גרביטציה במרחקים קטנים, וזאת בניגוד לידוע לנו מתוצאות ניסויים.

    אז איך זה מסתדר?

  111. אלבנטזו,
    אם יש לך סבלנות, אזי יש לי שני נושאים שאשמח לשמוע את דעתך.
    האחד מדבר על הפוסט הבא של שון קארול (איתך הסליחה שאני כל הזמן מביא מדבריו):
    http://www.preposterousuniverse.com/blog/2014/05/05/squelching-boltzmann-brains-and-maybe-eternal-inflation/

    בזמנו דיברנו על תוכנית בי בי סי שהציגה כמה רעיונות לגבי מה קרה לפני המפץ הגדול ואחד מהם היה של רוג׳ר פנרוז שאמר שכאשר הכל ידעך היקום ״יאבד״ את ממדיו ואז יכול להיווצר מחדש בגין פלקטואציה קוונטית (אני מקווה שאני לא טועה כאן!). האם המאמר של קארול ועמיתיו בעצם פוסלים את הרעיון הזה? האם בעצם וואקום, ללא צופה, אינו בלתי יציב? מה זה בעצם צופה? האם צופה הוא כל *חומר* שהוא קלאסי?
    זה גם מתקשר ל- ״ייקום מכלום״ של לורנס קראוס והוקינג. נשמע שקארול פוסל את הרעיון הזה.

    הנושא הבא הוא לגבי וענה פרופ. ג’יימס גייטס אמר בסימפוזיון נובל וגם בלינק המצורף לגבי error correction codes הקיימים בתורת המיתרים.
    http://youtu.be/cvMlUepVgbA
    מכיר את הנושא?

    תודה מראש

  112. יהודה,
    צר לי יהודה אבל הצעות הפתרון שלך מאוד לא רציניות, אם להיות עדין.
    TeVeS היא ניסיון לייצר משהו יחסותי אבל שלחתי אותך לשתי הרצאות בהן מספרים לך שזה עדיין לא פותר את הבעיה.
    ניסים מספר לך שלפי המודל הסטנדרטי (שהוא מוצלח בצורה מהממת בלהיות תואם למציאות) חלקיקי הפושינג גרוויטי לא יכולים להיות חלקיקים מוכרים ולפי וויקי הם גם צריכים להיות מהירים מהאור (אבל במהירות סופית כלשהי, מה סותר את היחסות שרציתי שכבודה יישמר) ואתה עונה בצורה מאוד חפיפניקית. זה לא שקשה להבחין בהם אלא שלפי המודל הסטנדרטי הם לא קיימים. אם הם כן קיימים, הם או מאוד מאוד מאוד כבדים ולכן תחום הפעולה שלהם מאוד מאוד מאוד קצר או שהם לא מייצרים כמעט אינטראקציה עם חומר רגיל ולכן לא יכולים לבצע את מה שאתה רוצה שיבצעו.
    אגב, לחלקיקים שמאוד קשה להבחין בהם קוראים חומר אפל

  113. ניסים

    לא מסכים עם ההנחות שלך:
    הם בנויים מחלקיקים בעלי תנע ואנרגיה קינטית.
    הם זעירים מאוד וקשה להבחין בהם.
    והרעיון של הפושינג גרביטי לא פוסל את תורת היחסות.

    לילה טוב הולך לישון
    ביי מגיבים מכל רחבי העולם!
    ביי יקום!
    תודה “ידען”!
    ()

  114. אמנם המודלים הפיסיקלים מסתדרים יפה עם מתמטיקה, אך נכון לעכשיו טרם קיבלתי צ’ובה לשאלה מה לפיסיקה ולחשבונות מכולת. כלומר, כיצד קורה שהמתמטיקה משחקת תפקיד במודלים האלה.
    הדתיים שבינינו (מזל שעכשיו שבת והם לא יחללו אותה כדי להרביץ בי ולהרביץ לי) מסתדרים יפה עם עולם דואלי בו מתמטיקה והשגחה שמיימית דרות בכפיפה אחת. אני, לעומתם, מחפש תמיד את הפתרונות הפשוטים. לא מוצא חן בעיני, למשל, שיְקוּמֵנו מאוכלס בכל כך הרבה חלקיקים אלמנטרים שאין להם אב קדום משותף. אמנם הפיסיקאים המודרנים ניסו לפייסני במודל הקווארקים, אך גם שש הוא מספר גדול מדי לטעמי, מה גם שלכל אחד מהקווארקים קיים אנטי-קווארק וכבר הגענו לתריסר אבני בניין נפרדות.
    המודל שעליו אני שוקד כבר 40 שנה (בקרוב יומולדת) מדבר על חלקיק יסודי אחד בלבד אשר כבר בימיו הראשונים הוא קיבל את הצורה של מה שמכונה היום בשם “חומר אפל”.
    החומר האפל הוא המרכיב את החלקיקים הידועים לפיסיקה. הוא גם זה שיוצר את תופעת הגראביטציה מחד ואת האלקטרומגנטיות מאידך.
    זו תהיה שגיאה לייחס לחומר האפל תכונות פיסיקאליות מוּכָּרות, היות שהפיסיקה נובעת ממנו, וייחוס כזה יהווה הגדרה מעגלית. לפיכך, הדיבור על “פוטון אפל”, למשל, הוא פשוט קשקוש ואני מאמץ בחום את הסיפא של המאמר “אָבֶּל 3827 יהפוך לעוד דוגמה למה שהחומר האפל איננו”.

  115. ישראל שפירא
    לא יהיה שום כוח משיכה באוויר כי הדרך החופשית הממוצעת של החלקיקים (המולקולות) בו היא קטנה מאוד ולכן לא תיווצר משיכה. אבל בתנאי ריקנות גבוהה אפשר יהיה לערוך ניסוי שיראה גרביטציה כי אז הדרך החופשית הממוצעת גדלה. אתה יש מבין ישראל?, אז מי רוצה להשקיע בניסוי?

  116. יהודה
    לפי הבנתי, הם לא יכולים להיות בנוים מחומר רגיל, כי אז הם היו מופיעים במודל הסטנדרטי.

    אני גם חושב שזה שלא מחפשים אותם, לא מחייב שלא היו מוצאים אותם.

    ודבר אחרון – הרעיון פוסל את שתי תורות היחסות. לא נראה לי שצריך לפסול כל כך מהר כלים שמזמן עברו מתחום המדע לתחום ההנדסה.

  117. יודה

    אילו היתה קיימת משיכה במנגנון גזי עם התנגשויות בין החלקיקים, היינו מקבלים משיכה בין כל שני סדינים מתוחים הפונים זה אל זה, לא?

    שים לב שללא התנגשויות, אתה כן מקבל משיכה כמו שמראה הערך על לאסז’.

  118. לישראל שפירא

    למה קבעת שבמנגנון גזי לא נוצרים כוחות משיכה???, אני כבר שנים חותר לעשות ניסוי שיראה כוחות משיכה בגז. מחפש מתנדבים לערוך ניסוי כזה.
    בנוסף לה סאז’ לא התייחס על אפשרות להתנגשות החלקיקים ביניהם כי אז הוא היה מתקן את ריבוע המרחק למרחקים גדולים ממש כמו שאני תיקנתי. ככל הידוע לי, רק אני העליתי רעיון זה .
    ודרך אגב הרעיון לחלקיק שליח שגורם לגרביטציה כבר הועלה על ידי ניוטון. הוא טען שחייב להיות כזה אבל הוא לא יודע איך זה עובד.

    יהודה

  119. ניסים
    החלקיקים האלה הם זעירים ובעלי מסה קטנה יותר מהניטרינים. יש להם תנע ואנרגיה קינטית והם קרוב לוודאי בנויים מחומר רגיל. הם כניראה כמו נטרינים קטנים. אני קורא להם חלקיקיי לה סאז’. לא מגלים אותם כי לא מחפשים אותם. אין רבים שמאמינים בפושינג גרביטי. אבל אם אני צודק בהעלמות הגרביטציה במרחקים גדולים זה יכול להיות הוכחה לקיומו של הסבר פושינג גרביטי לגרביטציה.

    לשמוליק
    אני לא חסיד של תיאוריית ה MOND אבל העידוש מתבצע ממעבר בין אזורים בעליי צפיפות גזית שונה שקיימת בסביבת הגלקסיה ולאו דווקא בגלל קירבה של מסה גדולה. אני יודע שקרן אור עושה זאת כמו בפטה מורגנה או מיראז’ אבל לא יודע איך זה מתבצע בדיוק.
    בנוסף אני לא דיברתי על חלקיקים מהירים ממהירות האור ולא דיברתי על תורת הייחסות. כל מה שאני חושב על תורת הייחסות שהיא צריכה לעבור תיקון למשל לגבי הגרביטציה במרחקים.
    האמת, לא התעמקתי בזה יותר מידי. מצידי , כבודה של תורת הייחסות נישמר. הסבר שונה של גרביטציה לא צריך לפגוע בבסיסם של מסקנות היחסות.
    אפשר לפרט ולפרט אבל, כבר אחרי חצות אז נא להגיב בעדינות. השכנים ישנים.
    יהודה

  120. יודה

    יובל חייקין גר בזמנו בסקוטלנד, עכשיו בניו ג’רזי. הוא זה שקישר אותי אליך.

    תיקון קטן: בלסאז’ פרופר אין התנגשויות בין החלקיקים, אחרת היית מקבל מנגנון גזי, שכידוע אינו יוצר גרביטציה.

    אני מבין את הטענה שלך שעל פי המודל שלך ההתנגשויות, root mean square, הן במרחקים גדולים ולכן אנו כן מקבלים מנגנון גזי, אך כפי שכבר ציינתי בפניך, במקרה כזה לא היית מקבל גרביטציה במרחקים קצרים, מה שסותר תוצאות ניסויים.

  121. יהודה,
    אז הפרש לחצים זה כרטיס יציאה מהכלא? אפשר פשוט לשלב אותו בשיחה מבלי להיות מומחה במה שנמדד ב bullet cluster?

    אם היוטיוב של קארול לא סייע, אולי ויקי יסייע?
    https://en.m.wikipedia.org/wiki/Bullet_Cluster

    In theories without dark matter, such as Modified Newtonian Dynamics (MOND), the lensing would be expected to follow the baryonic matter; i.e. the X-ray gas. However, the lensing is strongest in two separated regions near (possibly coincident with) the visible galaxies. This provides support for the idea that most of the mass in the cluster pair is in the form of two regions of dark matter, which bypassed the gas regions during the collision. This accords with predictions of dark matter as only weakly interacting, other than via the gravitational force.

    עדיין אני לא מבין איך הפרס לחצים יכול להסביר תוצאה זו

    אני גם לא מבין מדוע הצורך בחלקיקים הנעים במהירות גבוהה בהרבה מהאור אינה בעיה. איך אפשר להסביר אפקטים יחסותיים דרך התיאוריה הזו? מדוע הזמן זז לאט יותר כשטסים במהירות גבוהה? מדוע הזמן זז לאט יותר תחת כבידה? איך מוסבר פריים דרגינג?

  122. לניסים
    אתה מכריח אותי לפרט ובכן, הפושינג גרביטי יוצר גרביטציה על פי ריבוע המרחק בצורה ממש יפה ריצ’רד פיינמן מסביר זאת יפה בהרצאתו באוניברסיטת קורנל. פרטים בגוגל. אלא שיש עם זה שתי בעיות הראשונה היא החיכוך שביגללה ביטל ריצ’רד פיינמן את ההסבר . ואני טוען שזה כלום כי זה דורש תיקון קטן בנוסחת ניוטון שבו : כוח הגרביטציה פחות כוח החיכוך ייתן את הכוח הצנטרפוגלי. ממש כמו ספינות המפרש בים בהם כוח הרוח פחות החיכוך ייתן את כוח התנועה של ספינת המפרש.
    הבעיה השנייה היא שככל שהמרחק בין הגופים מתרחק קיימת יותר אפשרות שהחלקיקים מתנגשים בינם לבין עצמם , וכיוונם הופך לאקראי ולכן מאבד את מידע הגרביטציה הטמון בכיוון תנועתו. לכן אין גרביטציה (כמעט) במרחקים הקוסמולוגיים הגדולים. להסבר נוסף, אולי אכתוב מאמר על הפושינג גרביטי לאבי ואולי הוא יועיל בטובו לפרסמו.

    נא להגיב בעדינות. זה רק מדע.
    שבת שלום
    סבדרמיש יהודה

  123. ישראל שפירא, אתה בארה”ב???… חשבתי הרבה יותר קרוב…. לא גמרתי לבכות שעה. כניראה שהסיכוי שניפגש הולך ומתרחק ביחוד שגם היקום כולו מתפשט בתאוצה עם האנרגיה האפלה!!!!!!!!!!!!!

  124. אריאל לא טוען שיש פגם לוגי מהותי בכבידה של לסאז’, אלא פגם לוגי מהותי בתהליך הסקת המסקנות של יהודה. הוא נתן קישור לסרטון שבו הוא (כביכול, כמובן) מגיע בשיקולים לוגים למסקנה שהחומר החסר בגלקסיות ספירליות צריך להיות מוסבר על ידי הפרשי לחצים. הסרטון הזה עמוס בשטויות, בהנחת המבוקש, ובשגיאות לוגיות מביכות. אריאל ציין ספציפית את העובדה שמחד המסקנה של יהודה היא שצריך להחליף את כוח הכבידה – שמה שאנו יודעים עליו נכון רק למרחקים קצרים ולכן למעשה יתכן שהוא בכלל אחר לגמרי ממה שאנחנו חושבים – אבל מאידך הוא פוסל את האפשרות שיש התערבות של כוחות גרעיניים כי הם פועלים רק לטווח קצר. עזוב שזו שטות מוחלטת שאנו מודדים גרביטציה רק במרחקים קצרים (ישנן תופעות גרביטציוניות מדודות גם בסקלות הגדולות ביותר של היקום), ברור שיש כאן הגיון מעוות. מצד אחד – הכוח החזק (שנמדד רק במרחקים קצרים על פני כדה”א) חייב לציית לחוקים האלה גם במרחקים ארוכים ולכן לא יכול להסביר את התופעה, מצד שני גרביטציה נמדדה רק במרחקים קצרים ולכן יתכן שהיא מתנהגת על פי חוקים אחרים ומסבירה את התופעה.

    ההגיון של יהודה מעוות לגמרי, לא יעזור בית דין. הוא גם מדבר מבלי להכיר את הקצה של הקצה של העובדות, וגם זה לא ישתנה (הוא גם מודה בכך בפה מלא, אלא שמשום מה הוא טוען שאין צורך להכיר את העובדות כדי לחקור את הנושא). בכבידה של לסאז’ אין בעיות לוגיות, יש רק את העניין הקטן של אי-התאמתה לתצפיות. עניין פעוט, אמנם, אבל בכל זאת משום מה פיזיקאים עושים ממנו ביג דיל…

  125. יהודה
    אז החלקיקים האלה נספגים – במה? למה הם הופכים? אם הם נספגים בחלל ריק, לא הגיוני שיספגו עוד יותר במעבר דרך חומר?

  126. ישראל אני מודע טוב מאוד לאיך הכוח החזק עובד אני רק מציג את הטענה שלי ליהודה כדי להציג לו את הפגמים בלוגיקה שבה הוא משתמש.

  127. יודה

    ארצות הברית..

    והן פרשו מהממלכה המאוחדת כבר ב1776..

    וממלכתו של אבי היא הידען, אני מדבר על הפייסבוק. שלחתי לך בקשה לפני חודשיים..

    והכוח החזק לא שווה הרבה מחוץ לגבולות האטום. למעשה אם אני זוכר נכון, בשלב מסויים הוא מתהפך ונהפך לכוח דוחה במקום מושך..

    מזכיר לך משהו בהקשר של המאמר?

  128. יהודה אני רק טענתי פי ההיגיון שלך(אם לא מדדנו במרחקים גלקטיים אז זה לא בהכרח נכון) שיש סיכוי לא רע שזה דווקא כוח אחר(3/4) שמשפיע על תנועת הגלקסיות ולא בהכרח כוח הכבידה. אני רוצה לדעת מהם הטיעונים שלך למה זה חייב להיות כוח הכבידה ולא כוח אחר.

  129. לשמוליק
    הפרש לחצים יכול להזיז גלקסיות בצביר בלי צורך במסה אפלה או בכלל גרביטציה.
    התמונות היפות שמראות כאילו את המסה האפלה בצביר הקלע , מראות למעשה רק את אזוריי אי ההתאמה עם נוסחת הגרביטציה. הפרשי לחצים יכולים להסביר כל מה שמסה אפלה יכולה להסביר. מעבר לזה אינני מומחה בצביר הקלע.
    יום טוב
    סבדרמיש יהודה

  130. לאריאל

    אם תוכיח לי שהכוח החזק ו/או מפלצת הספגטי המעופפת ו/או המלאך גבריאל פועלים גם למרחקים גדולים, אשמח להשתמש בהם. סתם להגיד זה לא מקובל עלי. באופן עקרוני מכיוון שגבריאל וספגטי הם לא רעיונות מדעיים, אני מבטל אותם על הסף. (אי אפשר להראות דרך שאפשר להעמיד אותן בפני הפרכה.)

    לישראל שפירא היקר והאהוב

    חלילה לי לעשות מעשה נבלה כזה ולא לאשר את היקר בידידיי בממלכה המאוחדת. אם למישהו יש סמכות שלא לאשר אותך זה אבי היקר . כניראה רשמת איזה שהיא מילה “אפלה” שלא עברה את תוכנת הצנזור שלו. אבל הנה הנה הופעת. רק שתזכור שאני כל בוקר מחמם מחדש את הקפה שהכנתי לך ובדמעות בעיניי , אני, הכופר, מתפלל שאולי תופיע סוף סוף.
    ולדבריך אני, חובב הפושינג גרביטי, ולדעתי היא הסבר מקסים לגרביטציה, ונא לא להזכיר את החיכוך כי דווקא מראה את גדולתם של חלקיקיי לה סאז’. שיודעים להתגבר על החיכוך וליצור כוח גרביטציה. ממש כמו שספינות מפרש ששטות בים למרות החיכוך שיש להן עם המים והאטמוספרה. זאת לחלוטין אינה סתירה!!!
    דיון ממצה על דעתי בפושינג גרביטי הוא לא לעיניינו של מאמר זה.

    לניסים

    אתה צודק שיש מנגנון “ספיגה” אבל הוא ירחיב את הויכוח בעינייני הפושינג גרביטי שגם מצב תיאוריה זו הוא בהכחשה שוטפת בגלל דברי המתחרים ניוטון ריצ’ארד פיינמן ואחרים.
    אם האחראי באתר יסכים אני אכתוב מאמר בנושא. אבל הובהר לי שהוא לא מעוניין במאמרים שלא עומדים בקונצנזוס המדעי. וכמובן הוא הבוס.

    אז נא להגיב בעדינות
    זה רק מדע
    סבדרמיש יהודה

  131. לאריאל

    אם תוכיח לי שהכוח החזק ו/או מפלצת הספגטי המעופפת ו/או המלאך גבריאל פועלים גם למרחקים גדולים, אשמח להשתמש בהם. סתם להגיד זה לא מקובל עלי. באופן עקרוני מכיוון שגבריאל וספגטי הם לא רעיונות מדעיים, אני מבטל אותם על הסף. (אי אפשר להראות דרך שאפשר להעמיד אותן בפני הפרכה.)

    לישראל שפירא היקר והאהוב

    חלילה לי לעשות מעשה נבלה כזה ולא לאשר את היקר בידידיי בממלכה המאוחדת. אם למישהו יש סמכות שלא לאשר אותך זה אבי היקר . כניראה רשמת איזה שהיא מילה “אפלה” שלא עברה את תוכנת הצנזור שלו. אבל הנה הנה הופעת. רק שתזכור שאני כל בוקר מחמם מחדש את הקפה שהכנתי לך ובדמעות בעיניי , אני, הכופר, מתפלל שאולי תופיע סוף סוף.
    ולדבריך אני, חובב הפושינג גרביטי, ולדעתי היא הסבר מקסים לגרביטציה, ונא לא להזכיר את החיכוך כי דווקא מראה את גדולתם של חלקיקיי לה סאז’. שיודעים להתגבר על החיכוך וליצור כוח גרביטציה. ממש כמו שספינות מפרש ששטות בים למרות החיכוך שיש להן עם המים והאטמוספרה. זאת לחלוטין אינה סתירה!!!
    דיון ממצה על דעתי בפושינג גרביטי הוא לא לעיניינו של מאמר זה.

    לניסים

    אתה צודק שיש מנגנון “ספיגה” אבל הוא ירחיב את הויכוח בעינייני הפושינג גרביטי שגם מצב תיאוריה זו הוא בהכחשה שוטפת בגלל דברי המתחרים ניוטון ריצ’ארד פיינמן ואחרים.
    אם אבי בליזובסקי יירצה אני אכתוב מאמר בנושא. אבל אבי הבהיר לי שהוא לא מעוניין במאמרים שלא עומדים בקונצנזוס המדעי. וכמובן הוא הבוס.

    אז נא להגיב בעדינות
    זה רק מדע
    סבדרמיש יהודה

  132. יודה, לא נראה לי שהמגיבים מכירים את מנגנון הפושינג גרביטי..

    כדאי אולי רק לציין שהרעיון נבדק עיי כל הגדולים, כולל ניוטון, מקסוול, לורד קלווין, פיינמן ועוד רבים.

    ולמה אתה לא מאשר אותי בפייסבוק? שכחת כבר מי הגיש לך תורכי קטן במילואים?

    שמעתי גם שלסאז׳ וצוויקי הקימו קבוצת ווטסאפ, אז לא להתחמק.

  133. יהודה
    כלומר אתה מניח שהמרחב סופג את החליקיקים שמעבירים את כוח הכבידה, נכון? זה די סותר את מה שאנחנו מניחים שהמרחב לא סופג כלום, נכון?
    אם אתה חושב כך, אתה צריך לתאר את המנגנון של הספיגה.
    אתה גם צריך לסתור את שאר העדויות לחומר האפל.

    בהצלחה….

  134. יהודה, אז שוב תזכיר לי למה אתה לא חושב שזה לא הכוח החזק שמחליט פתאום לפעול אחרי אחרי מרחק של מיליארדי שנות אור?

  135. לניסים
    לכל תופעה שמתפשטת במרחב ככדור כמו גרביטציה גלי קול וגלי אור וכו, יש שני גורמים שמשפיעים על ההתפשטות. א. מה שכולם מכירים שזו ההתפשטות הכדורית שגם אתה מדבר עליה בצדק, אבל בנוסף ישנה ההפרעה של התווך לדוגמא, אור מגדלור מתפשט על פי ריבוע המרחק אבל גם על פי עכירות האטמוספרה. עכירות שתבלע 50 אחוז כל ק”מ (בגלל ערפל למשל) תותיר מהאור בקושי פרומיל ממה שצפוי על פי ריבוע המרחק.
    אני טוען שכנ”ל לכל תופעה המתפשטת בצורה ריבועית כולל הגרביטציה השאלה היא עבור איזה מרחק יוותר 50% מהגרביטציה. המצב כרגע עם ניוטון שמרחק זה הוא אין סופי כלומר כל הגרביטציה היוצאת לדרך מגיעה ליעדה. אני טוען מחישובים שעשיתי שמדובר על שנות אור בודדות שכבר יותירו רק 50% מהגרביטציה

    לאריאל
    תגובתך השניה
    יפה אמרת שאפשר היה לטפל במקום בגרביטציה , בנוסחה המעגלית שהיא למעשה החוק השני של ניוטון F=M*a
    שבמעגל הוא R/F=M*V2
    אבל זה בדיוק רעיונו של פרופ’ מילגרום בתיאוריית ה MOND שלו, הוא ביצע זאת ללא הצלחה ייתרה. יש תופעות שתאוריה זו מתקשה להסביר.
    כלומר, מה שאני עושה לגרביטציה דומה למה שעשה פרופסור מילגרום לחוק השני. המסקנות שונות.

    לאריאל
    תגובתך הראשונה
    אני מתאר לעצמי שמערכת שמש בגלקסיה רחוקה שהיא בגדלים דומים למערכת השמש שלנו תתנהג באותו אופן כמו זו שלנו כלומר על פי חוקי הגרביטציה של ניוטון למרחקים קטנים.
    בנוסף, אני מאמין שגם גלקסיה רחוקה תתנהג לגבי מרחקים גדולים של אלפי ועשרות אלפי שנות אור כמו הגלקסיה שלנו אבל שתיהן לא תעשינה זאת לא על פי חוקי הגרביטציה של ניוטון אלא על פי חוקים אחרים או מתוקנים.
    העיקרון הקוסמולוגי לא יכול להגדיל את טווח ההוכחה של נוסחאות רק ניסוי ומדידות יכולים לעשות זאת!

    נא להגיב בעדינות
    סבדרמיש יהודה

  136. יהודה
    מאחורי משוואת הכבידה יש היגיון מאד פשוט – שטח ההשפעה של מקור קרינה הומוגני תלוי בשטח כדור, שתלוי בריבוע הרדיוס. אם נסתכל על אור, עוצמת הקרינה תלויה במספר הפוטונים ליחידת שטח ליחידת זמן.
    גם זה, לדעתך, שגוי?

  137. יהודה, אם אתה זורק את העיקרון הקוסמולוגי לפח(שזה מה שעשית בתגובה האחרונה שלך) אתה עדיין צריך לענות על השאלות הבאות:
    1) מהם חוקי התנועה שפועלים בגלקסיות מרוחקות?(הרי חוקי התנועה של ניוטון תקפים למיטב ידיעתנו רק ב10 יחידות אסטרונומיות)
    2)אם אין איזוטריות מדוע הטמפ’ של קרינת הרקע וצפיפות הגלקסיות זהה?
    3) האם גלקסיה מרוחקת שאני צופה בה ברגע זה מתנהגת לפי כוחות באיזור שלי או לפי הכוחות באזור שלה? למשל אם הכבידה שמה חזקה פי 10 האם היא מפעילה עליי כוח החזק פי 10 ממה שאני מפעיל עליה? או שאנחנו מפעילים כוחות שקולים זה על זה ואם כן מהו החוזק של אותו כוח?

  138. לאריאל
    אמרת: “אנחנו לא יכולים לגזור שמשהו בגדלים שהמערכת השמש יהיה תקף לכל היקום.” סוף ציטוט.
    זה נכון אבל לא אני אמרתי אמר את זה הפילוסוף האנגלי דיוויד יום וגם אחרים: “ממה שאתה מודד אתה יודע רק על מה שאתה מודד”. למשל, בדקת 1000 ספרים וראית שהם עשויים נייר והם מספרים את סיפורו של פינוקיו, אז המסקנה אינה יכולה להיות שכל הספרים ביקום עשויים נייר ומתעסקים בפינוקיו, אלא רק שיש 1000 ספרים לפחות שעשויים נייר ביקום ומתעסקים בפינוקיו. ויותר מזה, אם משהו נכון במערכת השמש לעשרה יחידות אסטרונומיות אורך הוא לחלוטין לא חייב להיות נכון גם לפי מיליון או מיליארד אורך, כלומר….. אם כדור הארץ סובב סביב השמש בגלל גרביטציה , אין זה אומר שגם הגלקסיות הספיראליות הגדולות פי מיליארד מסתובבות בגלל גרביטציה. ובבקשה לא לצטט לי את העיקרון הקוסמולוגי כי מה שהעיקרון אומר שבמערכת שמש אחרת הדומה למערכת שלנו הגרביטציה פועלת. מה, לעשות אבל זה מה יש!, מצטער.
    נא להגיב בעדינות, זה רק מדע (:))
    סבדרמיש יהודה

  139. יהודה למה קבעת גרביטציה? למה לא המשוואות שלנו שמתארות תנועה מעגלית? למה לא שהכוח האלקטרומגנטי יתחיל להשפיע גם על חלקיקים ניטרלים? לפי ההיגיון שלך כל הכוחות שגילינו וחקרנו במערכת השמש יכולים להיות הגורמים כמו שאמרת אנחנו לא יכולים לגזור שמשהו בגדלים שהמערכת השמש יהיה תקף לכל היקום.

  140. יהודה למה קבעת גרביטציה? למה לא המשוואות שלנו שמתארות תנועה מעגלית? למה לא שהכוח האלקטרומגנטי יתחיל להשפיע גם על חלקיקים ניטרלים? לפי ההיגיון שלך כל הכוחות שגילינו וחקרנו במערכת השמש יכולים להיות הגורמים כמו שאמרת אנחנו לא יכולים לגזור שמשהו בגדלים שהמערכת השמש יהיה תקף לכל היקום.

  141. יהודה, מי הגאון “המוכשר” שצילם את ההרצאה שלך ביוטיוב ? איכות פשוט זוועה הייתי מתבייש להעלות דבר כזה לאינטרנט.

  142. לעובד
    קודם כל אני באמת שולל את הגרביטציה כמעט לחלוטין במרחקים הקוסמולוגיים הגדולים. היא אף פעם לא הוכחה במרחקים הקוסמולוגיים הגדולים. ניוטון מגלה הגרביטציה טען שהיא מוגדרת על כל היקום ומאז זה כך. אבל היקום של ניוטון היה קטן, רק מערכת השמש עד כוכב הלכת שבתאי, כעשרה יחידות אסטרונומיות. אז לו היה מותר להגיד שהנוסחה פועלת על כל היקום. לנו אסור להניח זאת על מיליוניי ומיליארדי שנות אור.
    ולגבי הפרש לחצים הבה ונהרוג עוד פרה קדושה ונישאל את עצמינו האם היקום שלנו ריק או שהוא מכיל אין ספור חלקיקים שנעים בתוכו כגון נטרינים קרניים קוסמיות אחרות, פוטונים וכו. כל אלה מגדירים גז כלומר היקום שלנו הוא גוף גזי אדיר המכיל בתוכו כוכבים. ולגז יש לחצים, הפרשי לחצים, רוחות וכו. טענתי שאחרי שביטלתי את כל האפשרויות האחרות נותר רק פתרון הפרש הלחצים בעימות עם רעיון המסה האפלה. וכמובן אסור לשכוח את העידוש, את האנרגיה האפלה ואת התפשטות היקום המואצת, הכל צריך להיות מוסבר.
    זה בקיצור ואם תרצה הסבר יותר ממצה נסה למצוא אותי ולהתקשר אלי ואני בכייף אשמח להיפגש ולהסביר לך ולאחרים את רעיונותי צריך רק לבוא עם ראש פתוח ובלי דיעות קדומות. למשל אם אמרת: “והאמת לא יכול לראות פיתרון שאינו כולל אפקטים גרביטציונים ,” סוף ציטוט, זו כבר החלטה שתצטרך להיות מוכן לוותר עליה.
    נא להגיב בעדינות
    סבדרמיש יהודה

  143. יהודה ראיתי את הסרטון ולא באמת הבנתי מה התאוריה שלך אומרת “השוואת הלחצים” ? זה לא אומר הרבה.. אולי תוכל להסביר מעבר לכך.. חוץ מזה שאני באמת לא מבין.. אתה בעצם שולל את התאוריה הגרביטציונית? זה מה שאתה אומר? כי נראה לי שכדאי שתחדד קצת את מה שאמרת שם במצגת , אני אישית ללא מכיר לומק את בעיית המסה האפלה מלבד העובדה שיש שם סתירה במשוואות שנובעות ממשואת התנועה (אך מה שאני מכיר טיפה יותר ארוך מהבעיה כפי שהצגת אותה פשטנית כל כך.. ) והאמת לא יכול לראות פיתרון שאינו כולל אפקטים גרביטציונים , בגדול מה שאתה אומר שם קצת לא ברור, אם אפשר אולי שתקשר למקום שמציג את מה שאתה מדבר עליו שצורה יותר אקדמית ומבוססת.. תודה

  144. לא הבנתי מדוע יש סתירה לכאורה. אם מישהו מכיר מאמר מדעי שמסביר את הסתירה, אשמח לראות.

    מי שרוצה לראות את הסברו של קארול לגבי טבס ו ה bullet cluster ומדוע משחק עם גרוויטציה ללא חומר אפל לא יכול להסביר את המידע שהגיע מוזמן לצפות החל מהדקה החמישים ביוטיוב הבא
    (כבר הבאתי אותו בעבר אבל יהודה מתעקש לא לראות או להבין…)
    http://youtu.be/iu7LDGhSi1A

  145. אני מצרף קישור לתאוריה מקורית
    שפורסמה לפני 12 שנים…

    היא פותרת את הצורך בחומר אפל…

    רק תצטרכו להסתגל למחשבה שמה שמכונה
    “חור-שחור-סופר-מסיבי” מסוגל לפלוט חומר…

    אגב
    הרבה מהקישורים חיצוניים לתמונות השתנו במשך 12 שנים

    http://www.mish-pat.com/assil2/assil2.htm

  146. למר/גב’ לא רוצה בחזרה למאה ה 16
    מה תעשה במאה ה16?, ניוטון עדיין לא נולד, וגליליאו עוד לא בנה טלסקופ. אולי תזכה לראות את ג’ורדנו ברונו עולה על המוקד אבל לא בטוח. וזה גם לא מחזה מלבב שכדאי לחזור בעבורו למאה ה16.
    מה שכן, אם אני הייתי אז, אין ספק שהייתי שקט מאוד ומשתדל לא לעצבן את האינקוויזיציה כדי שלא יבער לי התחת באנרגיות גבוהות שמישהו היה מארגן לי…
    אני חייב לציין שגם המשך תגובתך מעניינת אותי ועניין הניטרינו ומהירותו, מטריד אותי לא פעם. סופר נובה 1987A התפוצצה בענן המגלני הגדול לפני 170,000 שנים וראה זה פלא הנטרינים של הפיצוץ ואור הפיצוץ מגיעים אלינו כמעט ביחד זה לכאורה סתירה ליחסות !
    כך שהיה לנו על מה לדבר אם היינו ניפגשים ואלבנזו לא היה מפריע. אולי היינו מזמינים גם אותו?….ואת הגננת אסתר מגן במבי??.
    זה היה יכול להיות נחמד.
    לילה טוב
    יהודה

  147. כרגע פורסם באתר שחתן פרס ישראל פרופסור יעקב בקשטיין הלך לעולמו. שמעתי אותו מרצה על המסה האפלה ותיאוריית ה MOND. וגם הוא רואיין לא פעם בנושא. הוא היה מגדוליי החוקרים בארץ.
    עצוב.

  148. יהודה,
    אבל אתה שומע מפיזיקאים כולל מכאלו שעוד לא התנצחת מולם, שישנה עדות מכריעה לכך שלא משנה כיצד תשחק עם הגרוויטציה, ישנם ממצאים שלא יוכלו להיות מוסברים ע״י משחק כזה. קארול מסביר זאת על רקע הממצאים מ ה- Bullet Cluster, במפורש ובאנגלית קלה, ביוטיוב שהבאתי, ואתה בשלך. אני ממש לא מצליח להבין מדוע אתה מתעקש. על סמך איזה ידע אתה סותר את הטענה שקארול טען? על סמך אילו ממצאים?

  149. אריאל
    אכן הבהרת את הנקודה שלך היטב ובל נשכח שבסופו של דבר אלבנזו הוא השופט והוא הקובע ויש לו קשרים באיגוד השופטים. זה מה יש.

    שמוליק
    בגלקסיות הספירליות הישוו את הגרביטציה לכוח הצנטריפוגלי וגילו שהכוח הצנטריפוגלי הוא פי עשרה מכוח הגרביטציה. מכיוון שהם אמורים להיות שווים ביניהם הוסיפו את המסה האפלה. גדולתו של פרופסור מילגרום שהוא לקח את הנתונים הקיימים שנמדדו בשטח, ולא שינה אותם, אבל הוא כן שינה את הנוסחה של הכוח הצנטריפוגלי- החוק השני של ניוטון- חוק התאוצה F=m*a כך שהכוח הצנטריפוגלי יהיה פי עשרה יותר קטן ואז יהיה שוויון. ברגע שהוא ניכשל. הפתרון שנישאר הוא לפעול בכיוון שינוי נוסחת הגרביטציה.
    ערב טוב ואיזה כיף שיש מזגן בחדר!
    יהודה

  150. יש מוטיב חוזר מענין אצל כל מכחישי המדע למינהם.
    הטיעון הזה של – בדקתם את זה רק למרחקים או זמנים קצרים לכן זה יכול להיות שגוי…
    חוקי הפיזיקה אמורים להיות אוניברסלים . אותם שדות/כוחות עם משוואה יחידה שמתארת אותם.
    זה שלא בקרתי בהונולולו, לא אומר שאני לא יכול להניח שפועלים שם אותם חוקי פיזיקה.
    נראה לי שעל זה מבוסס כל ענף הקוסמולוגיה.
    למרות כל זאת …אני מוכרח להודות שסברמיש מהווה בידור קליל לצפיה. ושגם לי יש כל מיני הרהורי כפירה לפעמים על הימצאות בוזון ההיגס. וניטרינו מהירים ממהירות האור.
    נא לא להגיב בעדינות.

  151. יהודה יתכן ואתה צודק ויתכן שאתה לא,אבל בינתיים אל תשכח שאתה במרומי היציע והשופט אלבנצו נמצא על המגרש, יכול להיות שאתה רואה טעות שיפוט או “פאול” ואלבנצו לא שורק אבל אל תשכח שאלבנצו הוא זה שעל המגרש ולא אתה, אתה לא יודע אם הדשא רטוב,לא מכוסח וכו’… ואלבנצו דווקא כן יודע. מובן שאתה יכול לקבל את הנתונים האלה ואז להחליט אם היה “פאול” או לא אבל עד שאתה לא מקבל אותם אתה לא יכול לערער על החלטות השופט. בנוסף אתה גם אוהד של אותה קבוצה לכן שיקול הדעת שלך משוחד לעומת השופט שהוא יותר ניטרלי. מקווה שהבהרתי את הנקודה שלי.

  152. יהודה,
    MOND היא לא הסיבה שהמלצתי לך לדבר איתו (שוב, במקרה שלך) אלא שינסה הוא להסביר לך מדוע אף אחד לא משנה נתונים (כלומר אף אחד לא משנה דאטה שמגיע מהטלסקופים שלנו וכו׳). אף אחד לא יודע אם MOND שגויה אלא שגם אם היא טובה, היא לא נפתרת מהחומר האפל.

    בנוסף, חבל שהפסקת לשמוע מאחר ובהרצאה השניה, בערך בדקה ה- 40, הוא מסביר מדוע ה- Bullet Cluster מאתגר לכדי אבסורד כל תיאוריה שמנסה להסביר את התוצאות רק ע״י משחק עם גרוויטציה. חבל שהגעת לשוקת וסרבת לשתות ממנה. היה מחדש את האנרגיות האפלות החסרות שלך

  153. לשמוליק
    לפני שנים כבר יצא לי לשוחח עם פרופ מילגרום על תיאוריית ה MOND. פגישה מעניינת. לצערי היא לא עומדת במבחן ההוכחה . לדוגמא היא לא מסוגלת להסביר את התנהגותה של גלקסיה M94 שאין בה כמעט חומר אפל.

  154. לשמוליק
    ניגשתי ללינק ששלחת אותי אליו, והבנתי שהמרצה מנסה לשכנע שתיאוריית ה MOND אינה פיתרון למה שניקרא בעיית המסה האפלה ומכיוון שאני מסכים עם מר מרצה, לא מצאתי לנכון להמשיך להאזין להרצאתו. בימים טרופים אלה שאני כבר הרבה מעבר לגן “במבי” כדאי לי לנצל בצורה המיטבית את מעט האנרגיות הגבוהות שנותרו לי.
    לידיעתך, מלבד תיאוריית ה MOND ורעיון החומר האפל, יש לפחות עוד עשרה אפשרויות אחרות לפתרון הבעייה הקוסמולוגית הנ”ל. בכל אופן , תודה.
    נא להגיב בעדינות
    יהודה

  155. ממתינה אולי בגלל שכתבתי את האימייל של מילגרום (מהאתר שלו במכון ויצמן) כדי שיהודה ישאל אותו באופן ישיר על החומר האפל

  156. יהודה,
    ראית את ההרצאות שהפניתי אותך אליהם?
    המצב שאתה מתאר: “אתה כניראה לא מבין שהחומר האפל הוא שינוי הנתונים שנעשה כדי לעזור לחומר הרגיל להשלים את 90 האחוזים שחסרים לו להשלמת הגרביטציה החסרה לו” הוא פשוט הכפשה של כמעט כל העוסקים בתחום הזה ואני ממש לא מבין מדוע אתה מגיע למקומות הללו.

    יש לי אתגר בשבילך: חנן השיג לנו את האתר של מילגרום. הנה:
    http://www.weizmann.ac.il/particle/milgrom/
    זה האימייל שלו: [email protected]

    מה דעתך לשלוח לו מייל ובו אתה שוטח את טענותך כנגד חומר אפל (ואם אתה אמיץ כותב את המשפט שפתחת בו את הפוסט האחרון שלך)? תשובתו תהיה מאוד מעניינת

  157. לאלבנזו
    בסעיף 3 בתגובתך ביקשת:
    “תן דוגמא אחת לכך שפיזיקאי אי-פעם שינה נתונים שנמדדו בשביל לתמוך בחומר אפל”. סוף ציטוט.
    אתה כניראה לא מבין שהחומר האפל הוא שינוי הנתונים שנעשה כדי לעזור לחומר הרגיל להשלים את 90 האחוזים שחסרים לו להשלמת הגרביטציה החסרה לו. שינוי אווילי!, מי שלא מבין זאת, חבל! צר לי ידידי אלבנזו שאתה מזלזל כך בגן הילדים האהוב “במבי” ובמכללת שנקר. לא יפה!

    לאריאל
    חלילה לי מלפגוע בידע הרב שיש לאלבנזו באנרגיות גבוהות, נמוכות, ו/או , בחצי התורן, מה שאמרתי לאלבנזו שכאן מדובר על : הדברים הבסיסיים ביותר במדע כגון, מתי יש להחליף רעיון מדעי, והאם נוסחה (למשל הגרביטציה) שהוכיחה את עצמה למרחק של פחות מאלפית שנת אור (מערכת השמש הפנימית) תהיה חייבת להיות נכונה גם למרחקים של מיליוני שנות אור?. את זה לימדה אותי הגננת דוויד -יום בגן “במבי”. אלו דברים בסיסיים ביותר והידע הרב שאלבנזו מנפנף בו , הידע האדיר שלו בפיזיקה , אינו שייך לדברים הבסיסיים הנ”ל ואינו מוסיף להבנת הדברים, וכנראה אפילו גורע. אז כמובן שכל אחד יכול לדבר על החומר האפל או על גן הילדים האהוב “במבי” אבל היציבות המנטאלית של ילדי גן הילדים “במבי” האהוב, אינה קשורה למצב האנרגיות הגבוהות ביקום הקוסמולוגי. אז נא לדייק ולא להגיד שאני אמרתי שאסור לו!
    בשיעור הראשון במכללת שנקר (או ב”גן במבי” אני כבר לא זוכר כל כך טוב) אמר המורה למינהל (או הגננת?) שאתם יכולים להיות מאוד חכמים ומאוד מצטיינים אבל קחו בחשבון שתמיד יבוא אדם מן הישוב או מהגנון השני וימצא לכם טעויות. זה ממש דומה לחובב כדורגל של ביתר ירושלים המביט ממרומי היציע במשחק כדורגל עם מיטב השחקנים ומגלה טעויות של השופט אלבנזו, במשחק הכדורגל של ביתר ירושלים . “לה פמיליה” מוכיחים זאת בכל משחק.

    אך, איזה יום יפה ונעים!
    אז יש לי מצב רוח טוב אז נא להגיב בעדינות ולא לדכא אותי.
    יהודה.
    בוגר גאה של מיכללת שנקר וגן “במבי”.
    אוהד דווקא של מכבי.

  158. יהודה,
    שתי תגובות. אחת המקורית שלי
    עוד לגבי MOND ו- TeVes
    הנה הרצאה של שין קארול בה הוא מדבר על רעיונות כיצד להתקדם מעבר ליחסות כללית. בערך בדקה 31:20 הוא מתחיל לדבר על MOND ו- Teves ובערך בדקה ה- 40 הוא מגיע לפואנטה מדוע שני הרעיונות הללו לא מצליחים להיפטר מהחומר האפל.

    http://youtu.be/SwyTaSt0XxE

  159. “אני בוגר גן “במבי” (בהצטיינות יתרה), ראיתי את הסרט שליחות קטלנית 2, ויש לי נסיון של יותר 30 שנה בלהתקלח בעצמי בלי עזרה של אבא או אמא. לעניות דעתי הידע הזה מספיק והותר לניתוח מבנה הרשתות הנוירולוגיות במוח. למעשה, אני לא מבין למה לא נותנים לי לבצע הליכים כירורגיים בחולי סרטן מערכת העצבים…”

    אלבנצו, הרגת אותי…

    יהודה, הסיפא של אלבנצו עשוייה אולי להיראות בפניך מאוד פוגענית אבל המסר העולה ממנה הוא די מדוייק: ההשכלה שלך כפי שאתה מתאר אותה בפתיחות רבה, לחלוטין – אבל ממש ממש לחלוטין – אינה מספיקה להתמודד עם שאלות שהן בשלושה סדרי גודל יותר פשוטות משאלת החומר האפל.

    אדם צריך לדעת מה הם הכלים שיש לו באמתחת. לא היית יוצא לקרב כנגד גדוד צנחנים חמוש מכף רגל ועד ראש כשאתה מצוייד במקל. לא היית מנסה לפתור את בעית חיזוי מזג האוויר כשאתה מצוייד באריתמתיקה של כיתה ה’. נושא החומר האפל מצריך עומקים ואופקים שלוקח שנים ארוכות לבנות אותם, ואין פה קיצורי דרך.

    וגם היה ראוי שתתיחס להערות נקודתיות: חומר ולא מסה. לחומר עשויות להיות תכונות רבות, אחת מהן היא מסה. מודל או תאוריה ולא נוסחא. בפיזיקה של המאה ה-21 לא “ממציאים” נוסחאות אלא גוזרים את הנוסחאות המתאימות מתוך מודלים או תאוריות שמבוססים על עקרונות יסוד מאוד מוצקים ומאוד כוללניים. “נוסחא” היא רק עוד פינה במודל. ואחרי כל זאת אם החישובים והניבויים המתקבלים מהמודל אינם עולים בקנה אחד עם התצפיות הם מייד מוחלפים או ננטשים (או שאולי צריך לבצע כיול פרמטרים מחודש או למצער דפורמציה מבנית) עד שמתקבלת התאמה המקובלת על כל הקהילה החוקרת. אף אחד לא מנסה “להחליק” דברים, נהפוכו: הסטנדרטים של המחקר בפיזיקה הם גבוהים מאוד.

    אבל, כאמור, כדי להיווכח בזה צריך להמריא הרבה הרבה יותר גבוה מרמת לימודי התיכון או הנדסת תעשייה ושום קיצורי דרך לא יועילו. ללא ההמראה האופק נשאר צר מבלי שהמביט בו יודע שהוא צר.

  160. יהודה, מצד אחד אתה טוען שפיזיקאי שעוסק באנרגיות גבוהות לא יכול לדבר בכלל על גרביטציה אבל כמה פסקאות אחרי אתה רושם שאתה יכול לדון בנושא בגלל שאתה בוגר מכללת שנקר בניהול תעשייתי(שלמיטב ידיעתי אין שום קשר לכבידה). איפה העקביות שלך?!

  161. ודבר אחד ששכחתי להתייחס אליו – מטרת המאמר. מטרת המאמר היא לספר לך על תופעה חדשה שמתחילים לראות לה עדות נסיונית, של כוחות שפועלים בין חלקיקי חומר אפל. לצורך העניין, כוחות בסקטור האפל, או כוחות אפלים. זו המטרה, וכדי להשיג מטרה זו יש גם פסקה או שתיים של רקע. בחלק הרקע מסבירים על החומר האפל וגם על הקשיים בתיאוריה. אם תקרא מה כתוב במקום לסתום את האוזניים ולצרוח, תראה שכל הקשיים שמדברים עליהם הם בהבנת התכונות המיקרסקופיות של החומר האפל. כלומר, למרות שאנו רואים חומר אפל בריכוזים גבוהים בחלל, אין לנו חלקיק בודד ביד שנוכל לבצע עליו ניסוי ולגלות את מסתו, המטענים שלו, האינטראקציות שהוא מבצע וכו’. לצורך העניין, גם לניוטרינו יש מסה שאיננו יודעים מה היא בדיוק. אז לשיטתך הוא לא קיים, נכון?

  162. יהודה,

    להלן התייחסות תמציתית יחסית. לפני זה, ארצה לברך אותך על כך שעשית עבודה *מדהימה* (!!!, אי אפשר להדגיש זאת מספיק) בלהתעלם מכל אחד ואחד מהדברים שאמרתי. אני יכול להסביר זאת רק על ידי מקרה חריף בטירוף של הדחקת המציאות.

    1. הסברתי לך כבר שבמדע אין משמעות לרושם שנובע מהתנסחויות ובחירת מילים של אדם כזה או אחר, במיוחד כשאדם זה הוא עיתונאי ולא מדען מהתחום. תמשיך להתלות ב”תחושת יאוש” שיש לכתבת (אתה מבין את ההבדל בין כתב ומדען? לא בטוח), כי אין לך שום דבר אחר להאחז בו.

    2. הידע בפיזיקה של אנרגיה גבוהות לא שייך לנושא? קודם כל, אתה לא יודע איזה ידע יש לי. הידע שלי לא מסתכם רק באנרגיות גבוהות. תודה לאל, אני גם מכיר, עובד וכותב ביחסות כללית, שהיא כידוע גבול באנרגיות נמוכות בכלל של כבידה קוונטית. אבל זה לא משנה, כי – איך אתה יכול להחליט מה רלוונטי ומה לא? איך בנאדם שאין לו שמץ של מושג מהי התיאוריה של חומר אפל, אין לו שמץ של מושג מהי גרביטציה (מעבר לציטוט נוסחא שלומדים בכיתה ט’), יכול להחליט מה רלוונטי ומה לא? קודם תלמד את התחום, ואז אם אתה רוצה אני מזמין אותך לבחון אותי ואת ידיעותי, ואם הן רלוונטיות או לא. אני אתן לך רמז: אני פיזיקאי שמתעסק בגרביטציה. גרביטציה, יעני, כבידה.

    3. תן דוגמא אחת לכך שפיזיקאי אי-פעם שינה נתונים שנמדדו בשביל לתמוך בחומר אפל. אני באמת לא יודע אם אתה שקרן או שאתה פשוט לא מבין מה ההבדל בין מדידה ובין רעיון שבא לך בחלום. הסברתי בתגובה קודמת – אף אחד לא משנה את הנתונים, משנים את התיאוריה. החומר האפל הוא תולדה של שינוי תיאוריה (בשפתך הילדותית – שינוי הנוסחא. פשוט במקום לשנות את המעריך של המרחק בנוסחא כמו שאתה רוצה לעשות בלי להבין בכלל מה זה אומר, שינינו את המקורות של שדה הגרביטציה בנוסחא).

    4. צריך “מדען עם ביצים” כדי “לזרוק את הרעיון הגרביטציוני”? לא. צריך מישהו ממש טיפש. כמה טיפש? אני אתן לך דוגמא:

    5. “אני בוגר מכללת שנקר בניהול תעשייתי בהצטיינות וניהלתי עסק שעסק בייעול מפעלים למעלה מארבעים שנים די בהצלחה. לעניות דעתי הידע הזה מספיק והותר לצורך ניתוח בעיית החומר האפל.”.
    זהו. פשוט אין מה להוסיף. בנאדם שהוא מספיק טיפש כדי להגיד כזה דבר, וגם מספיק טיפש בשביל לא להבין כמה זה טיפשי, באמת לא ראוי לדיון. אני בהחלט מסכים שאפשר לחתוך כאן.

    אה, רגע – דבר אחד אחרון. אני בוגר גן “במבי” (בהצטיינות יתרה), ראיתי את הסרט שליחות קטלנית 2, ויש לי נסיון של יותר 30 שנה בלהתקלח בעצמי בלי עזרה של אבא או אמא. לעניות דעתי הידע הזה מספיק והותר לניתוח מבנה הרשתות הנוירולוגיות במוח. למעשה, אני לא מבין למה לא נותנים לי לבצע הליכים כירורגיים בחולי סרטן מערכת העצבים…

  163. יהודה,
    גם אני ניסיתי באופן בלתי מתערב להראות לך, דרך הרצאה מאוד נחמדה של שון קארול שהנושא שלה הוא ״מעבר ליחסות כללית״, מדוע קיים חומר אפל. ההרצאה באנגלית קלה ומוסבר שם היטב, בדקות שציינתי מדוע מילגרום לא הצליח להיפטר מהחומר האפל. טרחת לראות את ההרצאה?

    שאלת מה מטרת המאמר וכאן נכשלת כישלון חרוץ בהבנת טקסט והגדלת בכך שעשית צ׳רי פיקינג בכך שהצגת את הפיסקה הראשונה כחזות המאמר. אוי ואבוי. הפיסקה הראשונה דרמתית כי מטרתה לנסות להכניס את הקורא לעניין והיא מסתיימת בסימן שאלה, כלומר, היא מבקשת ממך להמשיך לקרוא.

    האם הגעת לפיסקה השלישית? (או השניה, תלוי איך אתה סופר)? היא נפתחת במילים הבאות:
    ״בתסריט העגום הזה חלה תפנית מעוררת תקווה בתחילת אביב 2015. חוקרים חשפו את אחד הרמזים המסקרנים ביותר זה שנים: סימן לקיומו של כוח חדש שעשוי לאפשר לחומר האפל “לדבר” עם עצמו. התובנה הזאת תעזור להסביר מאיזה סוג חלקיקים עשוי, אולי, החומר האפל.״

    מה לגבי הפיסקה הזו:
    ״החדשות האלה ריגשו את הפיזיקאים ועוררו אותם לחפש תשובות. “אם התגלית הזאת תחזיק מעמד, אז זה הרבה מעבר לחדשות מרעישות,” אומר הפיזיקאי ניל ויינר מאוניברסיטת ניו יורק, שלא נטל חלק במחקר. תרחיש הכולל פוטונים אפלים הוא שינוי של התפיסה הבסיסית והמקובלת ביותר של חומר האפל כמורכב מסוג אחד ויחיד של חלקיקים, שמקובל לכנותו “חלקיק מסיבי המנהל אינטראקציות חלשות”, או בראשי תיבות WIMP (“חלשלוש”, באנגלית). אבל הרעיון שלחומר אפל מתלווים פוטונים אפלים ואינטראקציות אקזוטיות עשוי לעזור לפתור כמה בעיות שמעורר תיאור החומר האפל באמצעות WIMP מסוג אחד, כמו למשל מדוע מרכזי הגלקסיות פחות דחוסים מן הצפוי.״

    בהמשך המאמר החוקרים עשו מה שאתה מצפה מכל חוקר ראוי וזה לתת את הדיסקליימרים הראויים, שכן לא מדובר בדת אלא במדע וכמה יפה מצד כותבת המאמר להכניס את הציטוטים הללו למאמר.

    אז מה היה במאמר ומה מטרתו? לספר לנו על הממצאים האחרונים בתחום שמעוררים התרגשות לכל מי שעוסק בתחום. פוטונים אפלים, כוח בין חלקיקים אפלים ולא חלקיק אפל..,איזה יופי. מזמן לא קראתי מאמר שחידש לי כל כך הרבה והריץ אותי לוויקי. יישר כוח לידען!

    אז אני לא מבין איך אתה קורא מאמר כזה וכל מה שאתה לוקח ממנו זו את הפיסקה הראשונה שלו, שמסתיימת בסימן שאלה(!), כמטרת המאמר.

  164. יהודה
    ההבנה שלך על המאמר הפוכה משלי. המאמר אופטימי ולא טוען בשום מקום שרעיון החומר האםל שגוי

    יהודה – הגישה שלך לא מדעית להחריד. יש לך היפותיזה נוספת שפותרת את הבעיות שרעיון של חומר אפל פותר? לא נראה לי שזה כך. אתה מציע נוסחה שמתאימה לחלק מהתוצאות, ואינה מסבירה תופעות אחרות שהחומר האפל כן פותר (למשל, להבנתי, עידוש כבידתי).

    וכמו שאלבנצו אמר – אם הנוסחה שלך סותרת את תורת היחסות הכללית – אז נזרוק גם את זה?

    תהיה רציני….

  165. לאלבנזו
    ראה את ההבדל ביננו. אני קורא את הכתבה ורואה בה את הייאוש של הכותב בו הוא מודה שעד היום אין שום הוכחה למציאותה של המסה האפלה ומקווה הוא שמחקרו זה יביא איזה שהיא הוכחה.
    אתה לעומת זאת כבר בטוח מראש שהכול מוכח, החומר האפל הוא עובדה קיימת, ולמעשה אנשים כמו יהודה רק סתם מבלבלים את הראש. אז תגיד לי אלבנזו, מה מטרתו של המאמר הזה אם הכול מוכח בעסקי החומר האפל?

    דבר נוסף שאני רואה כאן הוא הדברים הבסיסיים ביותר במדע כגון, מתי יש להחליף רעיון מדעי, ואם נוסחה (למשל הגרביטציה) שהוכיחה את עצמה למרחק של פחות מאלפית שנת אור (מערכת השמש הפנימית) תהיה חייבת להיות נכונה גם למרחקים של מיליוני שנות אור. אלו דברים בסיסיים ביותר והידע הרב שאתה מנפנף בו , הידע האדיר שלך בפיזיקה , למשל של אנרגיות גדולות, וכו’, אינו שייך לדברים הבסיסיים הנ”ל ואינו מוסיף להבנת הדברים, וכנראה אפילו גורע.
    לכן, עם כל הערכה שלי לידע שלך, הוא לא שייך לנושא כאן.

    וכשאני קורא לרעיון החומר האפל ביקום רעיון הזוי אני רוצה שמי שקורא את תגובתי יבין את דעתי עליה, יבין שמה שנוצר כאן זה לא מסה או חומר. מה שנוצר כאן הוא שינוי הנתונים הנמדדים בשטח כדי שיתאימו לנוסחה או כמו שאתה אומר “לרעיון המדעי הקיים”, שינוי שאסור שיעשה והוא נוגד את הכלל הבסיסי ביותר במדע הדן בשאלה: מתי יש לזרוק רעיון מדעי קלוקל.

    אלבנזו, זרוק את הרעיון הגרביטציוני, הוא לא מתאים למרחקים גדולים.
    ונכון, צריך להיות מדען עם ביצים כדי לעשות מעשה כזה של ביטול הגרביטציה כמעט לחלוטין למרחקים הקוסמולוגיים הגדולים.

    לסיום לצערי, אני כנראה לא הגיב לך יותר. פשוט נמאסה עלי ההתייחסות השחצנית שלך כלפי, ניבולי הפה שלך וכינויי הגנאי המעליבים. אני בוגר מכללת שנקר בניהול תעשייתי בהצטיינות וניהלתי עסק שעסק בייעול מפעלים למעלה מארבעים שנים די בהצלחה. לעניות דעתי הידע הזה מספיק והותר לצורך ניתוח בעיית החומר האפל. אין צורך בידע הרב שלך.

  166. יהודה,
    אני מנסה להבין… בהנחה שיש לך, לדעתך, מודל חלופי שמסביר את התופעות שבנדון, האם ניסית לפרסם אותו בעיתון מדעי? האם ניסית לחבור לפיזיקאי שתומך במודל שלך ומוכן לעזור לך לפרסם את הדברים? אתה מחזיק בדעות האלה כבר שנים, ודאי ניסית לעשות עם זה משהו, נכון?

  167. יהודה,

    1. כן, אני מהבודדים. אתה יודע, אני ועוד כמה שיושבים פה לידי במסדרון של המחלקה ל-high energy physics. ועוד כמה פיזיקאים מהאוניברסיטה הסמוכה. ועוד כמה בודדים מהאוניברסיטה שלידה. וגם אלה שבשאר ארה”ב, וגם אלה שבאירופה, ואסיה, וישראל.. אתה יודע, מיעוט קטנטן כזה של 99% מהפיזיקאים. וגם החלק הזערורי של פיזיקאים שחושבים שאין חומר אפל, כמו מילגרום, לא יסכימו איתך. כי הם – בניגוד אליך – טרחו ללמוד במה מדובר. בדקנו את העדויות, את הראיות, את התצפיות, מכירים את המודלים התיאורטיים. מילגרום מבין שלא משנה מה כתוב בספר לתלמידי תיכון בני 15 – בפיזיקה אין נוסחאות קסם. אי אפשר סתם להחליט שבמקום המרחק בחזקת מינוס שתיים תשים מינוס שתיים וחצי. כל הקשרים נובעים מהמודל, במקרה שלנו יחסות כללית, ואם אתה רוצה לשנות את ה”נוסחא”, אתה צריך להמציא תורת גרביטציה חדשה לגמרי. והוא מנסה כבר 30 שנה לעשות את זה, ועל אף ההערכה שיש לי אליו (גילוי נאות: אני מכיר אותו באופן אישי), הרעיון שלו פשוט לא עובד. כבר עשרות שנים שהוא או עומד במקום, או חוטף מכות מתצפיות שפשוט מראות שהוא לא נכון (כמו ה-bullet cluster שהזכרתי מקודם, וראיתי שגם שון קארול מדבר עליה בהרצאה ששמוליק הפנה אליה).

    2. כמובן, אין לך שום דבר תוכני להגיד, אז אתה פונה לכל מיני רשמים והתנסחויות של כתבת באתר הידען, שהיא כלל לא פיזיקאית מהתחום. אז מהניסוחים שלה אתה מקבל הרגשה שהיא לא בטוחה בקיום החומר האפל. מה איכפת לי? מדע לא עושים לפי “איזה רושם מקבל יהודה סבדרמיש מהניסוח של עיתונאית שמספרת על מדע (ולא עושה אותו בעצמה)”, אלא לפי מחקרים. ראיות, תצפיות, ניסויים, מתמטיקה. אתה יודע, כל הדברים האלה שאתה מפחד או מתעצל לנסות ללמוד. ברור, עדיף לסתום את האוזניים ולצעוק בקול גדול, “אין מסה אפלה!”.

    3. ואם כבר דיברנו על “מסה אפלה”, אז אני מסכים איתך ששמות זה לא מה שחשוב. אני מקווה גם שלא איכפת לך שבמקום מכניקת הקוונטים נדבר על “רעיון הקוונטוטות”, במקום יחסות כללית נלמד “יחסות גנרלית”, ובמקום מכניקה ניוטנית נחקור את “תורת אייזיק לענייני תנועה”. ברור שהשמות לא חשובים. אבל כשאתה ממציא שפה משלך, אתה סתם נשמע כמו אדיוט. ואם לדקדק – מה שקיים הוא החומר. חומר יכול להיות מאסיבי, ויכול להיות גם לא מאסיבי. זו תכונה של החומר. החומר הוא אפל, לא המסה.

    4. אז אני רק מנבל את הפה ואין לי תוכן לדברים, הא? מצחיק. על “הפוסל במומו פוסל” שמעת פעם? אתה לא כותב בתגובות שלך כ-ל-ו-ם, חוץ מלקרוא לי “מר מדען”, “אור המדע”, “מדען המדענים” שאלה כמובן ביטויים סרקסטיים שמטרתם לזלזל ביכולת המדעית שלי. אני, לעומת זאת, הצגתי מספר טיעונים קונקרטיים שאתה מתעלם מהם באופן מוצהר. למשל, שלא טרחת ללמוד את העדויות והתצפיות לתיאוריה שאתה פוסל. למשל, שהטענות שלך הן עובדתית לא נכונות (שינוי “נוסחת” הגרביטציה לא יפתור את הבעיה – להיפך, הוא יצור הרבה יותר בעיות כי כל תופעה מוסברת בגרביטציה תפסיק להיות מוסברת אם תשנה את תורת היחסות הכללית). למשל, שהטענה שלך בדבר זה שהפיזיקאים משנים את הנתונים במקום את התיאוריה היא פשוט שקר מוחלט. התגובה שלי הייתה מלאה בטיעונים עניינים. אבל מכיוון שאתה טיפש מדי/ פחדן מדי להתמודד איתם, אז אתה טוען שלא נתתי אף טיעון ענייני. נו, מילא.

    5. כמובן שאתה גם שקרן קטן, כי מעולם לא אמרתי ש”כל מי שחושב אחרת הוא כמובן חסר ידע , מטומטם, בור, וכו וכו וכו”. אני הסברתי פעמים רבות שאני מכבד מאוד את הנסיונות של אנשים למצוא תורות אלטרנטיביות והסברים שאינם כוללים חומר אפל. מה לעשות שהם נכשלים, בעוד חומר אפל רק צובר עוד ועוד ראיות לקיומו. וכמו שכבר אמרתי – יחי ההבדל הקטן בין פיזיקאי שלומד את הנושא ומנסה למצוא פתרון אמיתי לבעיה, ואיזה אדם ערירי שעשה קריירה אינטרנטית מלהכנס לפורומים מדעיים ולהפיץ מידע שגוי ושקרי בנוגע לחומר אפל. כמו שאמרתי לך, אני מכיר באופן אישי את האנשים שחוקרים תורות אלטרנטיביות ואני *מבטיח* לך שאם הם היו קוראים את התגובות שלך הם היו מזועזעים לפחות כמוני, אם לא יותר.

    6. אתה חושב שהמסקנות מהידע שלי לא נכונות? מאיפה לך לדעת? הרי אין לך מושג מה הידע שלי. לפחות 5 (לדעתי קרוב יותר ל-10) פעמים שניסיתי לשכנע אותך לקרוא על התיאוריה שאתה כל כך מתנגד לה. נתתי לך לינקים למאמרים שעברו ביקורת עמיתים על התיאוריה. נתתי לך לינקים למסמכים מדעיים שמונים את הראיות הפוזיטיביות לקיום חומר אפל. המלצתי לך על ספרים שסוקרים את הנושא. כמובן שמעולם לא טרחת ללמוד. אז אם אין לך שמץ של מושג מה אני יודע על חומר אפל, איך בדיוק אתה מחליט שהמסקנות שלי שגויות?

    אתה יכול להתרעם על הסגנון שלי. אתה יכול להתרפק על ניסים, שמסיבה כלשהי נוהג בך בכפפות של משי. אבל האמת המרה היא שאתה לא שונה משצף המגיבים הדתיים (והסתם פרנואידים-סכיזופרניים שחושבים שטכנולוגיית פלאש הגיעה מהחלל החיצון) ששולט באתר בחודשים האחרונים. כולכם סובלים בדיוק מאותה מחלה – החלטתם מה נכון ומה לא לפני שטרחתם לפתוח ספר או לבצע ניסוי. אתה מבין, מה שמילגרום עושה זה מדע. לוקח את הנתונים, המדידות, הראיות, מנסה לבנות מודלים מתמטיים שיספקו פרדיקציות, ולבחון אותם. מה שאתה עושה זה האנטיתזה של זה. לכן למרות ששניכם תומכים בהסבר לגרביטציה שאינו כולל חומר אפל, לא יכלתם להיות שונים יותר זה מזה. הוא מדען, ואתה מנסה להחזיר אותנו למאה ה-16. בהצלחה עם זה.

  168. יהודה, בעניין המינוחים עוד אפשר לוותר למרות שאני לא מבין מדוע אתה מתעקש להיות חוכם ולהשתמש במינוחים אישיים שלך במקום אלו שמקובלים על כולם.

    לגבי העניין עצמו, הוסבר לך כי ישנן תצפיות שנותנות אישוש חזק לתיאוריית החומר האפל, מדוע אתה מתעלם מהן ? או לחילופין, כיצד אתה מסביר אותן טוב יותר ללא החומר האפל ?

  169. יהודה, בעניין המינוחים עוד אפשר לוותר למרות שאני לא מבין מדוע אתה מתעקש להיות חוכם ולא להשתמש במינוחים המקובלים על כולם?

    אבל לגבי העניין עצמו, הוסבר לך כי ישנן תצפיות שנותנות אישוש חזק לתיאוריית החומר האפל, מדוע אתה מתעלם מהן או כיצד אתה מסביר אותן טוב יותר ללא החומר האפל ?

  170. יהודה, לגבי המינוחים עוד אפשר לוותר למרות שאני לא מבין מדוע אתה מתעקש להיות חוכם ולא להשתמש במינוחים המקובלים על כולם?

    אבל לגבי העניין עצמו, הוסבר לך כי ישנן תצפיות שנותנות אישוש חזק לתיאוריית החומר האפל, מדוע אתה מתעלם מהן או כיצד אתה מסביר אותן טוב יותר ללא החומר האפל ?

  171. לסברי מרנן
    עם כל הערכה שיש לי לידע של אלבנזו (ויש לי הערכה…. ואפילו מעט קינאה…) אני חושב שהמסקנות מהידע שלו הן “קצת” מוטעות ואני לא חייב להסכים איתן. ואיך לקרוא לרעיון המסה / החומר/ ההזוי/ האפל / הדמיוני/ הבידיוני / השקוף / זה באמת לא חשוב כל עוד שמבינים שהוא האסון המדעי הגדול ביותר של המאה העשרים !!!
    נא להגיב בעדינות, זה ס”ה מדע!
    סבדרמיש יהודה

  172. יהודה, קצת מצחיק שאחרי ש elbentzo מתקן אותך ומסביר לך שצריך להגיד ״חומר אפל״ ולא “מסה אפלה” אתה מתעקש להמשיך לקרוא לזה ״מסה אפלה״.

  173. יוני,
    אני מציע שתראה את היוטיוב שצרפתי. הוא מסביר בצורה לא רעה מה גילינו בצורה אמפירית (כולל ע”י מילגרום) והיכן נכנסות לתמונה MOND ו- TeVes

  174. יוני
    אלבנצו הסביר שוב ושוב – החומר האפל אינו “פתרון” שהומצא לבעיות הכבידה, אלא, יש הרבה תצפיות נוספות, שמאשרות שהחומר האפל אכן קיים.

    שינוי נוסחת הכבידה לא נותן מענה לשאר סוגי התצפיות, והוא מסביר שום דבר.

  175. לא צריך להעליב ..
    יהודה סובר כמו התיאוריה אלטרנטיבית שפיתח פרופ’ מוטי מילגרום ממכון ויצמן…
    http://www.themarker.com/magazine/1.2208993
    כל ההולדה של החומר האפל – נובע מההבדל שיש בין שתי השיטות חישוב של כמות המסה … זה שכולם הלכו לפתרון הראשון – לא מצדיק שהוא דווקא הנכון …

  176. עוד לגבי MOND ו- TeVes
    הנה הרצאה של שין קארול בה הוא מדבר על רעיונות כיצד להתקדם מעבר ליחסות כללית. בערך בדקה 31:20 הוא מתחיל לדבר על MOND ו- Teves ובערך בדקה ה- 40 הוא מגיע לפואנטה מדוע שני הרעיונות הללו לא מצליחים להיפטר מהחומר האפל.

    http://youtu.be/SwyTaSt0XxE

  177. יהודה
    איך זה ניתן להפרכה? כל מה שתעשה זה תשנה שוב את הנוסחה.
    מה שאני אומר זה שצאיך להסביר למה יש את התופעות המוזרות האלה.. מה שאתה עושה, לדעתי, שקול לאפיציקלים ולדפרנטים של תלמי. תוסיף מספיק – זה יעבוד, ואם לא יעבד אז נוסיף עוד אחד.

    זה עדיין לא מסביר כלום….

  178. יהודה

    אתה האדם היחיד שכותב כאן,
    אשר מטיל ספק בקביעות “מדעיות”.

    משום מה, לא מתנפלים עליך, כמו שמתנפלים עלי…

    אולי בגלל שאתה מאוד מאוד נזהר שלא לדרוך על יבלות…

  179. לאלבנזו
    דבר אחד אפשר להגיד עליך שאתה מהבודדים שמאמינים במציאותה של המסה האפלה בלי שום ספק. כל השאר כולל בעלי המאמר הנידון יש להם ספיקות כותב המאמר קורא למציאות המסה האפלה “תסריט עגום” הוא מתחיל את המאמר במילים:
    “יש משהו אי שם ביקום. אנחנו לא יכולים לראות אותו, אנחנו לא יכולים לגעת בו ואנחנו יודעים שהוא שם רק בזכות המשיכה הכבידתית שהוא מפעיל על העצמים ביקום. זה עשרות שנים, הסיפור של החומר האפל היה תגלית אחר תגלית בנוגע למה שהחומר המסתורי הזה איננו, סינון הדרגתי של אפשרויות שגרם לפיזיקאים להרגיש עצבנות גדלה והולכת. מה יקרה כשהמועמד האחרון יימחק מן הרשימה? האם ייחרץ גורלנו, לא לזכות לעולם להציץ בטיבו של החומר שתורם כ-25% מסך כל המסה ביקום?” סוף ציטוט.
    ניראה לי שגם כותב המאמר מבין ששמונים שנים של חיפושים הם די והותר והוא מסיים את המאמר במילים “לתחושתי, זה עכשיו או לעולם לא.”
    אבל לך מר מדען אלבנזו, אור המדע, מדען המדענים, אין שום ספיקות, המסה האפלה קיימת, בלי כל ספק בגלל כל מיני מילים מפוצצות שהכנסת לתגובתך. וכל מי שחושב אחרת הוא כמובן חסר ידע , מטומטם, בור, וכו וכו וכו. סלח לי, אבל אתה מבוזבז. תתחיל לחשוב מחוץ לקופסה ואולי תגיע למסקנות שלי.
    לצערי מלבד לזלזל ולנבל פיך על תגובתי לא אמרת כלום לגבי הספקות הקימות בנושא המסה האפלה כבר 80 שנה. לך אין כל ספקות. חבל!

    לניסים
    מדוע אתה קובע שנוסחת הגרביטציה אינה ניתנת להפרכה?, לידיעתך היא הופרכה בבדיקתה בגלקסיות הספירליות ובצבירי הגלקסיות. זה שמישהו החליט לשנות את הנתונים עם המסה ההזוייה זה לא אומר שהיא לא הופרכה.
    סתם, זו דעתי!
    נא להגיב בעדינות.
    סבדרמיש יהודה

  180. יהודה
    אגיד לך איך אני רואה את זה. אתה מסתכל על המדידות, מתאים להם נוסחה, והולך הבייתה.

    מה לזה ולמדע? אין פה מדע – זה לא מסביר כלום, זה לא מנבא כלום, וזה לא ניתן להפרכה.

  181. מתי יקומו המדענים ויעשו מה שאתה אומר? מתי שבאוניברסיטאות יפסיקו ללמד פיזיקה. מתי שמדענים יחליטו שלא מתאים להם ללמוד את החומר שהם חוקרים, שמספיק לקרוא ספר העשרה לתלמידי תיכון ולבסס עליו את כל הידע שלנו. בקיצור, כשכל הפיזיקאים יתחילו להיות ממש ממש מטומטמים.

    כרגיל, יהודה כותב ערימה של שטויות. אבל יש שטות אחת שחייבים להתייחס אליה באופן פרטי, ויש דבר אחד שהוא כותב שהוא דווקא לא שטות.

    השטות המיוחדת היא ש”מספיק לחפש את המסה ההזוייה”. מאמירה שטותית זו, בפרט מהמילים “מסה הזויה”, נובע שהיא לא קיימת. אז איך לבשר לך את זה, מצאנו אותה כבר. חומר אפל (מספיק כבר להגיד “מסה אפלה”, אף אחד חוץ מיהודה סבדרמיש לא קורא ככה לחומר האפל) נמצא כבר. יש תצפיות שלו בכל מיני מקומות בחלל. מה שעדיין אין זה חתיכה קטנה ממנו, כאן, על כדה”א, שמאפשרת לנו לחקור את התכונות המיקרוסקופיות שלו (כמו ההרכב שלו, מטענים תחת כוחות שונים וכו’). אבל החומר האפל בצבירים גדולים כבר נצפה במשך שנים. דוגמא יפה אפשר לראות, למשל, ב-bullet cluster.

    הדבר הנכון שאמר יהודה זה שכשמודל מתמטי לא מתאים למדידות, צריך לזרוק את המודל (בבקשה תפסיק להשתמש במילה “נוסחא”, כבר מאות שנים שאין “נוסחאות” בפיזיקה. יש מודלים מתמטים שמהם נובעים קשרים מסוימים, אבל אי אפשר לשנות את הקשרים בלי לשנות את המודל). ובאמת – פעם היה מודל שיש לו הרבה שמות, למשל FRW, וחשבו שהוא מצוין. ואז המשיכו עוד קצת, וגילו שהוא לא מתאים למדידות. המודל – שלא היה בו חומר אפל או אנרגיה אפלה – פשוט לא הסכים עם המציאות. אז זרקו אותו. חיפשו, חיפשו, ומצאו מודל חדש. המודל החדש התאים למדידות באופן פנטסטי. למודל החדש היו פרדיקציות שהתאמתו במעבדה. למודל החדש היו ראיות פוזיטיביות לנכונותו. למודל קראו LCDM, או בעברית – “למדא סי די אם”. למדא זו האות היוונית שמייצגת את האנרגיה האפלה, ו”סי די אם” זה ראשי תיבות של קולד דארק מאטר. יעני, חומר אפל קר. אבל יהודה רוצה שנזרוק את המודל הזה. למה? כי הוא לא מבין אותו… למעשה, הוא כל כך פוחד ממנו עד שכל פעם שמביאים לו לינק לסקירה של המודל, כולל הראיות לנכונותו (עם מדידות – לא מדובר במשהו תיאורטי אלא מאומת במעבדה), הוא מסרב לקרוא אותו באיזה תירוץ קלוש (“זה באנגלית”, “זה ארוך”, “אין לי זמן”…).

    יהודה, רוצה שיגיבו לך בעדינות? תפסיק לכתוב במשך שנים על גבי שנים את אותה תגובה מטומטמת שנובעת מבורות (אתה מדבר על דברים שאין לך מושג בהם ואתה מסרב באופן מוצהר ללמוד אותם) ומוציאה דיבתם של אנשים חכמים ממך פי 10, כנים ממך פי 100, וחרוצים ממך פי 1000 שאשכרה מנסים לקדם את ההבנה שלנו של היקום הסובב אותנו ולא להחזיר אותה אחורה לימי הביניים.

  182. בשיעור הראשון בתיכון בפיזיקה אמר המורה שאם התוצאות המתקבלות מנוסחה אינן מתאימות לנתונים הנמדדים בשטח אז את מי צריך לזרוק , את התוצאות הנמדדות בשטח או את הנוסחה?, לכולם ברור שצריך לעשות דבר פשוט והוא לזרוק את הנוסחה ולחפש אחרת. לכולם ברור, אבל לפעמים צריך קצת תעוזה. מתי יקומו מדענים אמיצים ויחליטו שזהו נוסחת הגרביטציה מיצתה את עצמה ואחרי 350 שנה מאז ימי ניוטון יש להחליפה באחרת. התנועה בגלקסיות חייבת להיות מוסברת בלי הגרביטציה הקדושה!, אין כל היגיון להדבק אליה, ולשנות את הנתונים עם חומרים הזויים כדוגמת חומר ואנרגיה אפלים שכל תפקידם הוא לשמר את נוסחת הגרביטציה במתכונתה. שמונים שנה חלפו מאז שפריץ צביקי העלה את רעיון המסה האפלה כדי לשמר את נוסחת הגרביטציה של ניוטון- איינשטיין גם במרחקים הגדולים של הגלקסיות וצביריי הגלקסיות. מספיק לחפש את המסה ההזוייה. הגיע הזמן לשנות.
    נא להגיב בעדינות!
    סבדרמיש יהודה

כתיבת תגובה

האימייל לא יוצג באתר. שדות החובה מסומנים *

אתר זה עושה שימוש באקיזמט למניעת הודעות זבל. לחצו כאן כדי ללמוד איך נתוני התגובה שלכם מעובדים.